Med Surg 2 Exam 5

¡Supera tus tareas y exámenes ahora con Quizwiz!

The patient has lateral angulation of the large toe toward the second toe. The nurse knows that what will be included in the treatment. a. metatarsal arch support b. trimming with a scalpel after softening c. surgery to remove the bursal sac and bony enlargement d. Interaarticular corticosteroids and passive manual stretching

c

*A nurse is reviewing the medical record of a client who has cystocele. Which of the following findings should the nurse identify as a risk factor for development of this disorder?* A. BMI of 18 B. Nulliparity C. Chronic constipation D. Postmenopausal

d

*Nursing care for the patient with endometrial cancer who had a total hysterectomy and salpingectomy and oophorectomy includes* a. maintaining absolute bed rest b. keeping the patient in high Fowler's position c. need for supplemental estrogen after removal of ovaries d. encouraging movement and walking as much as tolerated

d

*To decrease the patient's discomfort related to discussing his reproductive organs, the nurse should* a. relate his sexual concerns to his sexual partner b. arrange to have male nurses care for the patient c. give him written material and ask if he has questions d. maintain a nonjudgmental attitude toward his sexual practices

d

*A nurse is discussing gout with a client who is concerned about developing the disorder. Which of the following findings should the nurse identify as risk factors for this disease?* Select all that apply A. Diuretic use B. Obesity C. Deep sleep deprivation D. Depression E. Cardiovasular disease

A, B, E

*A nurse is providing information to a client who has osteoarthritis of the hip and knee. Which of the following information should the nurse include in the information?* Select all that apply A. Apply heat to joints to alleviate pain. B. Ice inflamed joints for 30 min following activity. C. Reduce the amount of exercise done on days with increased pain D. Prop the knees with a pillow while in bed E. Active range of motion is more effective than passive

A. Apply heat to joints to alleviate pain. C. reduce the amount of exercise done on days with increased pain E. active range of motion is more effective than passive

*A nurse working in an outpatient clinic is assessing a client who has rheumatoid arthritis (RA). The client reports increased joint tenderness and swelling. Which of the following findings should the nurse expect?* Select all that apply A. Recent influenza B. Decreased range of motion C. Hypersalivation D. Increased blood pressure E. Pain at rest

A. CORRECT: Exacerbating factors, such as a recent illness, are indicative of RA. B. CORRECT: Decreased range of motion is indicative of RA. C. INCORRECT: Xerostomia is indicative of RA. D. INCORRECT: Blood pressure changes are not indicative of RA. E. CORRECT: Pain at rest is characteristic of RA.

Which type of bone cell is responsible for the formation of bone? A. Osteocyte B. Osteoclast C. Osteoblast D. Sarcomere

Answer: C. Osteoblast Rationale: Osteocytes are mature bone cells. Osteoclasts are responsible for the reabsorption of bone. A sarcomere is the contractile unit of myofibrils.

*A nurse is teaching a client who has SLE about self-care. Which of the following statements by the client indicates an understanding of the teaching?* A. "I should limit my time to 10 minutes in the tanning bed." B. "I will apply powder to any skin rash." C. "I should use a mild hair shampoo." D. "I will inspect my skin once a month for rashes."

C. "I should use a mild hair shampoo." so it doesn't irritate the scalp

What is the most common diagnostic test used to assess musculoskeletal disorders? A. Myelogram B. Arthroscopy C. Standard X-ray D. MRI

C. Standard x-ray

The patient is diagnosed with torticollis. What should the nurse be prepared to provide for the patient? A. An immobilizer to hold the bones in place B. Exercises to increase the strength of the muscles C. A pillow to use to support the knew while sleeping D. Enough pillows to support the patient's head comfortably

D. Enough pillows to support the patient's head comfortably

A 78-year-old woman has a physiologic change related to aging in her joints. What is an appropriate nursing intervention related to comment changes of aging in the musculoskeletal system? A. Encourage rest to eliminate fatigue B. Provide all care for the patient to ensure that the care is completed C. Encourage eating enough calories to avoid the risk for impaired skin integrity D. Have the patient exercise to maintain muscle strength and avoid risk for falls

D. Have the patient exercise to maintain muscle strength and avoid the risk for falls

*A nurse in a provider's office is reviewing the medical record of a client who has fibrocystic breast condition. Which of the following findings should the nurse expect?* A. Palpable rubberlike lump in the upper outer quadrant B. BRCA1 gene mutation C. Elevated CA-125 D. Peau d'orange dimpling of the breast

a

*A nurse is assessing a client who has a casted compound fracture of the femur. Which of the following findings is a manifestation of a fat embolus?* A. Altered mental status B. Reduced bowel sounds C. Swelling of the toes distal to the injury D. Pain with passive movement of the foot distal to the injury

a

Patient-Centered Care: A patient is started on alendronate (Fosamax) once weekly for the treatment of osteoporosis. The nurse determines that further instruction about the drug is needed when what is said by the patient a. I should take the drug with a meal to prevent stomach irritation b. This drug will prevent further bone loss and increase my bone density c. I need to sit or stand upright for at least 30 minutes after taking the drug d. I will still need to take my calcium supplements while taking this new drug

a

*Postoperative care for a patient who had an abdominal hysterectomy include* Select all that apply a. monitoring urine output b. changing position frequently c. restricting all food for 24 hours d. observing perineal pad for bleeding e. encouraging leg exercises to promote circulation

a, b, e.

What is the best description of the periosteum? A. Lining of a joint capsule B. A characteristic of skeletal muscle C. Most common type of cartilage tissue D. Fibrous connective tissue covering the bone

d. Fibrous connective tissue covering bone Rationale: Synovium is the lining of the joint capsule. Striated is a characteristic of skeletal muscle. Hyaline is the most common type of cartilage

1. A 60-year-old woman has pain on motion in her fingers and asks the nurse whether this is just a result of aging. The best response by the nurse should include what information? a. Joint pain with functional limitation is a normal change that affects all people to some extent. b. Joint pain that develops with age is usually related to previous trauma or infection of the joints. c. This is a symptom of a systemic arthritis that eventually affects all joints as the disease progresses. d. Changes in the cartilage and bones of joints may cause symptoms of pain and loss of function in some people as they age.

1. d. Cartilage destruction in the joints affects the majority of those affected by the age of 40 and when the destruction becomes symptomatic, osteoarthritis (OA) is said to be present. Degenerative changes cause symptoms after age 50 or 60 but more than half over age 65 have x-ray evidence of OA. Joint pain and functional disability should not be considered a normal finding in aging persons. OA is not a systemic disease but is usually caused by a known event or condition that directly damages cartilage or causes joint instability.

10. Laboratory findings that the nurse would expect to be present in the patient with RA include a. polycythemia. b. increased immunoglobulin G (IgG). c. decreased white blood cell (WBC) count. d. anti-citrullinated protein antibody (ACPA).

10. d. The anti-citrullinated protein antibody (ACPA) is more specific than RF for RA and may allow for earlier and more accurate diagnosis. Other tests include C-reactive protein (CRP) that is elevated from inflammatory reactions of RA, a finding that is useful in monitoring the response to therapy. The white blood cell (WBC) count may be increased in response to inflammation and is also elevated in synovial fluid. Anemia, rather than polycythemia, is common, and immunoglobulin G (IgG) levels are normal.

11. Which other extraarticular manifestation of RA is most likely to be seen in the patient with rheumatoid nodules? a. Lyme disease b. Felty syndrome c. Sjögren's syndrome d. Spondyloarthropathies

11. b. Rheumatoid nodules develop in 20% to 30% of patients with RA. Felty syndrome is most common in patients with severe, nodule-forming RA. It is characterized by splenomegaly and leukopenia. Sjögren's syndrome occurs as a disease by itself or with other arthritic disorders. Lyme disease is a spirochetal infection transmitted by an infected deer tick bite. Spondyloarthropathies are interrelated multisystem inflammatory disorders that affect the spine, peripheral joints, and periarticular structures but they do not have serum antibodies.

12. Which drug that prevents binding of the tumor necrosis factor and inhibits the inflammatory response is used in the management of RA? a. Anakinra (Kineret) b. Entanercept (Enbrel) c. Leflunomide (Arava) d. Azathioprine (Imuran)

12. b. Entanercept binds to tumor necrosis factor (TNF) and blocks its interaction with the TNF cell surface receptors, which decreases the inflammatory response. Anakinra is an interleukin-1 receptor antagonist, thus decreasing the inflammatory response. Leflunomide is an antiinflammatory that inhibits proliferation of lymphocytes. Azathioprine is an immunosuppressant that inhibits DNA, RNA, and protein synthesis.

13. The patient has had RA for some time but has not had success with previous medications. Although there is an increased risk for tuberculosis, which monoclonal antibody is used with methotrexate to best treat symptoms? a. Parenteral gold b. Certolizumab (Cimzia) c. Tocilizumab (Actemra) d. Hydroxychloroquine (Paquenil)

13. b. Certolizumab is a monoclonal antibody that is a TNF inhibitor and stays in the system longer and may show a more rapid reduction in RA symptoms. Parenteral gold alters immune responses that may suppress synovitis of active RA but it takes 3 to 6 months to be effective. Tocilizumab blocks the action of the proinflammatory cytokine interleukin-6 (IL-6). Hydroxychloroquine is an antimalaria drug used initially for mild RA and requires periodic eye examinations to assess for retinal damage.

14. A 70-year-old patient is being evaluated for symptoms of RA. The nurse recognizes what as the major problem in the management of RA in the older adult? a. RA is usually more severe in older adults. b. Older patients are not as likely to comply with treatment regimens. c. Drug interactions and toxicity are more likely to occur with multidrug therapy. d. Laboratory and other diagnostic tests are not effective in identifying RA in older adults.

14. c. Because older adults are more likely to take many drugs, the use of multidrug therapy in RA is particularly problematic because of the increased likelihood of adverse drug interactions and toxicity. Rheumatic disorders do occur in older adults but usually in milder form. Older adults are not less compliant with drug regimens but may need help with complex regimens. Interpretation of laboratory values in older adults is more difficult in diagnosing RA because of age-related serologic changes but the disease can be diagnosed.

15. After teaching a patient with RA about the prescribed therapeutic regimen, the nurse determines that further instruction is needed when the patient says what? a. "It is important for me to perform my prescribed exercises every day." b. "I should perform most of my daily chores in the morning when my energy level is highest." c. "An ice pack to a joint for 10 minutes may help to relieve pain and inflammation when I have an acute flare." d. "I can use assistive devices such as padded utensils, electric can openers, and elevated toilet seats to protect my joints."

15. b. Most patients with RA experience morning stiffness and morning activities should be scheduled later in the day after the stiffness subsides. A warm shower in the morning and time to become more mobile before activity are advised. Management of RA includes daily exercises for the affected joints and protection of joints with devices and movements that prevent joint stress. Splinting should be done during an acute flare to rest the joint and prevent further damage.

16. A patient recovering from an acute exacerbation of RA tells the nurse that she is too tired to bathe. What should the nurse do for this patient? a. Give the patient a bed bath to conserve her energy. b. Allow the patient a rest period before showering with the nurse's help. c. Tell the patient that she can skip bathing if she will walk in the hall later. d. Inform the patient that it is important for her to maintain self-care activities.

16. b. Pacing activities and alternating rest with activity are important in maintaining self-care and independence of the patient with RA, in addition to preventing deconditioning and a negative attitude. The nurse should not carry out activities for patients that they can do for themselves but instead should support and assist patients as necessary. A warm shower or sitting in a tub with warm water and towels over the shoulders may help to relieve some stiffness.

17. After teaching a patient with RA to use heat and cold therapy to relieve symptoms, the nurse determines that teaching has been effective when what is said by the patient? a. "Heat treatments should not be used if muscle spasms are present." b. "Cold applications can be applied for 15 to 20 minutes to relieve joint stiffness." c. "I should use heat applications for 20 minutes to relieve the symptoms of an acute flare." d. "When my joints are painful, I can use a bag of frozen corn for 10 to 15 minutes to relieve the pain."

17. d. Cold therapy is indicated to relieve pain during an acute inflammation, can be applied with frozen packages of vegetables, and should last only 10 to 15 minutes at a time. Heat in the form of heating pads, moist warm packs paraffin baths, or warm baths or showers is indicated to relieve stiffness and muscle spasm. Heat should not be applied for more than 20 minutes at a time.

18. The nurse teaches the patient with RA that which exercise is one of the most effective methods of aerobic exercise? a. Ballet dancing b. Casual walking c. Aquatic exercises d. Low-impact aerobic exercises

18. c. The best aerobic exercise is aquatic exercises in warm water to allow easier joint movement because of the buoyancy of the water. Water produces more resistance and can strengthen the muscles. Tai Chi is also a good form of gentle, stretching exercise that would be appropriate. Dancing and walking impact the joints of the feet and even low-impact aerobics could be damaging. Exercises for patients with RA should be gentle.

19. A patient is seen at the outpatient clinic for a sudden onset of inflammation and severe pain in the great toe. A definitive diagnosis of gouty arthritis is made on the basis of what? a. A family history of gout b. Elevated urine uric acid levels c. Elevated serum uric acid levels d. Presence of monosodium urate crystals in synovial fluid

19. d. The diagnosis of gout is established by finding needlelike monosodium urate crystals in the synovial fluid of an inflamed joint or tophus. Hyperuricemia and elevated urine uric acid are not diagnostic for gout because they may be related to a variety of drugs or may exist as a totally asymptomatic abnormality in the general population. Although there is a familial predisposition to hyperuricemia, both environmental and genetic factors contribute to gout.

2. Number in sequence from 1 to 6 the pathophysiologic processes that occur in osteoarthritis (OA). ________ a. Erosion of articular surfaces ________ b. Incongruity in joint surfaces ________ c. Reduction in motion ________ d. Joint cartilage becomes yellow and granular ________ e. Osteophytes form at joint margins ________ f. Cartilage becomes softer and less elastic

2. a. 3; b. 5; c. 6; d. 1; e. 4; f. 2

20. During treatment of the patient with an acute attack of gout, the nurse would expect to administer which drug? a. Aspirin b. Colchicine c. Allopurinol (Zyloprim) d. Probenecid (Benemid)

20. b. Colchicine has an antiinflammatory action specific for gout and is the treatment of choice during an acute attack, often producing dramatic pain relief when given within 12 to 24 hours. Allopurinol, a xanthine oxidase inhibitor, is used to control hyperuricemia by blocking production of uric acid. Probenecid is a uricosuric drug that is used to control hyperuricemia by increasing the excretion of uric acid through the kidney. Aspirin inactivates the effect of uricosuric drugs and should not be used when patients are taking probenecid and other uricosuric drugs.

21. A patient with gout is treated with drug therapy to prevent future attacks. The nurse teaches the patient that what is most important for the patient to do? a. Avoid all foods high in purine, such as organ meats. b. Have periodic determination of serum uric acid levels. c. Increase the dosage of medication with the onset of an acute attack. d. Perform active range of motion (ROM) of all joints that have been affected by gout.

21. b. During therapy with probenecid or allopurinol, the patient must have periodic determination of serum uric acid levels to evaluate the effectiveness of the therapy and to ensure that levels are kept low enough to prevent future attacks of gout. With the use of medications, strict dietary restrictions on alcohol and high-purine foods are usually not necessary. When the patient is taking probenecid, urine output should be maintained at 2 to 3 L per day to prevent urate from precipitating in the urinary tract and causing kidney stones. Patients should not alter their doses of medications without medical direction and the drugs used for control of gout are not useful in the treatment of an acute attack. Joint immobilization is used for an acute attack of gout.

22. What characteristics are common in spondyloarthritides associated with human leukocyte antigen (HLA)-B27? a. Symmetric polyarticular arthritis b. Absence of extraarticular disease c. Presence of rheumatoid factor and autoantibodies d. High level of involvement of sacroiliac joints and the axial skeleton

22. d. An unusually high frequency of human leukocyte antigen (HLA)-B27 is found in patients with ankylosing spondylitis, psoriatic arthritis, and reactive arthritis and these diseases have a predilection for involvement of the spine, peripheral joints, and periarticular structures as well as an absence of rheumatoid factor and autoantibodies.

23. An important nursing intervention for the patient with ankylosing spondylitis is to teach the patient to do what? a. Wear roomy shoes with good orthotic support. b. Sleep on the side with the knees and hips flexed. c. Keep the spine slightly flexed while sitting, standing, or walking. d. Perform back, neck, and chest stretches and deep-breathing exercises.

23. d. Kyphosis and involvement of costovertebral joints in ankylosing spondylitis lead to a bent-over posture and a decrease in chest expansion, manifestations that are managed with chest expansion and deep-breathing exercises. Postural training emphasizes avoiding forward flexion during any activities and the patient should sleep on the back without the use of pillows.

24. Which descriptions are related to reactive arthritis (select all that apply)? a. Methotrexate is a treatment of choice b. Symptoms include urethritis and conjunctivitis c. Diagnosed by finding of hypersensitive tender points d. Increased risk in persons with decreased host resistance e. Infection of a joint often caused by hematogenous route f. Self-limiting arthritis following GI (enteral) or sexually transmitted infections

24. b, f. Reactive arthritis is self-limiting and follows GI or sexually transmitted infection, with symptoms including urethritis and conjunctivitis. Methotrexate is the treatment of choice for psoriatic arthritis. Hypersensitive tender points diagnose fibromyalgia. There is increased risk of septic arthritis in persons with decreased host resistance. Joint infection may be caused by the hematogenous route.

25. What is the pathophysiology of systemic lupus erythematosus (SLE) characterized by? a. Destruction of nucleic acids and other self-proteins by autoantibodies b. Overproduction of collagen that disrupts the functioning of internal organs c. Formation of abnormal IgG that attaches to cellular antigens, activating complement d. Increased activity of T suppressor cells with B-cell hypoactivity, resulting in an immunodeficiency

25. a. In systemic lupus erythematosus (SLE), autoantibodies are produced to nucleic acids, erythrocytes, coagulation proteins, lymphocytes, platelets, and many other self-proteins. This is a hypersensitive response, not immunodeficiency. Overproduction of collagen is characteristic of systemic sclerosis and abnormal IgG reactions with autoantibodies are characteristic of RA.

26. What is an ominous sign of advanced SLE disease? a. Proteinuria from early glomerulonephritis b. Anemia from antibodies against blood cells c. Dysrhythmias from fibrosis of the atrioventricular node d. Cognitive dysfunction from immune complex deposit in the brain

26. c. All body systems are affected by SLE. When the atrioventricular and sinus nodes are fibrosed and dysrhythmias occur, this is ominous. Although lupus nephritis can occur and lead to chronic kidney disease, treatment is available. Anemia, mild leukopenia, and thrombocytopenia are often present. Disordered thought processes, disorientation, memory deficits, and depression may occur.

27. A patient with newly diagnosed SLE asks the nurse how the disease will affect her life. What is the best response the nurse can give the patient? a. "You can plan to have a near-normal life since SLE rarely causes death." b. "It is difficult to tell because the disease is so variable in its severity and progression." c. "Life span is shortened somewhat in people with SLE but the disease can be controlled with long-term use of corticosteroids." d. "Most people with SLE have alternating periods of remissions and exacerbations with rapid progression to permanent organ damage."

27. b. Patients with SLE often find that one of the most difficult facets of the disease is its extreme variability in severity and progression. There is no characteristic pattern of progressive organ involvement, nor is it predictable as to which systems may become affected. SLE is now associated with a normal life span but patients must be helped to adjust to the unknown course of the disease.

28. During an acute exacerbation, a patient with SLE is treated with corticosteroids. The nurse would expect the corticosteroids to begin to be tapered when which serum laboratory results are evident? a. Decreased anti-DNA b. Increased complement c. Increased red blood cells (RBCs) d. Decreased erythrocyte sedimentation rate (ESR)

28. a. Efficacy of treatment with corticosteroids or immunosuppressive drugs is best monitored by serial serum complement levels and anti-DNA titers, both of which will decrease as the drugs have an effect. A reduction in erythrocyte sedimentation rate (ESR) is not as specific and the patient with SLE often has a chronic anemia that is not affected by drug therapy.

29. What should the nurse include in the teaching plan for the patient with SLE? a. Ways to avoid exposure to sunlight b. Increasing dietary protein and carbohydrate intake c. The necessity of genetic counseling before planning a family d. The use of nonpharmacologic pain interventions instead of analgesics

29. a. Acute exacerbations of SLE may be precipitated by overexposure to ultraviolet light, physical and emotional stress, fatigue, and infection or surgery. Dietary recommendations include small, frequent meals and adequate iron intake. Although SLE has an identified genetic association with HLA-DR3, genetic counseling is not a usual recommendation. The major concern in planning a pregnancy is that there are increased risks for the mother and fetus during pregnancy and exacerbations are common following delivery. Although nonpharmacologic methods of pain control are encouraged, the use of NSAIDs is often necessary to help control inflammation and pain.

3. What is most likely to cause the pain experienced in the later stages of OA? a. Crepitation b. Bouchard's nodes c. Heberden's nodes d. Bone surfaces rubbing together

3. d. The pain in later OA is caused by bone surfaces rubbing together after the articular cartilage has deteriorated. Crepitation occurs earlier in the disease with loose particles of cartilage in the joint cavity. Bouchard's nodes and Heberden's nodes are tender but occur as joint space decreases and as early as 40 years of age.

30. During assessment of the patient with scleroderma, what should the nurse expect to find? a. Thickening of the skin of the fingers and hands b. Cool, cyanotic fingers with thinning skin over the joints c. Swan neck deformity or ulnar drift deformity of the hands d. Low back pain, stiffness, and limitation of spine movement

30. a. Scleroderma is a disorder of connective tissue that causes skin thickening and tightening, resulting in symmetric, painless swelling or thickening of the skin of the fingersand hands, expressionless facial features, puckering of the mouth, and a small oral orifice. It does not cause the swan neck or ulnar drift deformities seen in RA or SLE. Low back pain and spinal stiffness are associated with ankylosing spondylitis.

31. When caring for the patient with CREST syndrome (calcinosis, Raynaud's phenomenon, esophageal dysfunction, sclerodactyly, and telangiectasia) associated with scleroderma, what should the nurse teach the patient to do? a. Maintain a fluid intake of at least 3000 mL/day. b. Avoid exposure to the sun or other ultraviolet light. c. Monitor and keep a log of daily blood pressure (BP). d. Protect the hands and feet from cold exposure and injury.

31. d. One of the most common and early manifestations of CREST syndrome (calcinosis, Raynaud's phenomenon, esophageal dysfunction, sclerodactyly, and telangiectasia) is Raynaud's phenomenon, which causes paroxysmal vasospasms of the digits with diminished blood flow to the fingers and toes on exposure to cold or stress, followed by cyanosis and then erythema on rewarming. The hands and feet must be protected from cold exposure and possible burns or cuts that might heal slowly and smoking is contraindicated. Sensitivity to ultraviolet light is not a factor in scleroderma, nor is fluid intake. Cardiovascular involvement may occur but it does not require patient monitoring.

32. During the acute phase of dermatomyositis, what is an appropriate patient outcome? a. Relates improvement in pain b. Does not experience aspiration c. Performs active ROM four times daily d. Maintains absolute rest of affected joints

32. b. Dermatomyositis produces symmetric weakness of striated muscle and weak neck and pharyngeal muscles may produce dysphagia. Weakened pharyngeal muscles lead to a poor cough, difficulty swallowing, and increased aspiration risk. Muscle tenderness or pain is uncommon, as is joint involvement. During an acute attack the patient is so weak that bed rest is needed and passive ROM is usually required.

33. During assessment of the patient diagnosed with fibromyalgia, what should the nurse expect the patient to report? a. Generalized muscle twitching and spasms b. Nonrestorative sleep with resulting fatigue c. Profound and progressive muscle weakness that limits ADLs d. Widespread musculoskeletal pain that is accompanied by inflammation and fever

33. b. People with fibromyalgia typically experience nonrestorative sleep, morning stiffness, irritable bowel syndrome, and anxiety in addition to the widespread, nonarticular musculoskeletal pain and fatigue. Fibromyalgia is nondegenerative, nonprogressive, and noninflammatory. Neither muscle weakness nor muscle spasms are associated with the disease, although there may be tics in the muscle at the tender points.

34. What is one criterion identified by the American College of Rheumatology for a diagnosis of fibromyalgia? a. Fiber atrophy found on muscle biopsy b. Elimination of all other causes of musculoskeletal pain c. The presence of the manifestations of chronic fatigue syndrome d. The elicitation of pain on palpation of at least 11 of 18 identified tender points

34. d. The American College of Rheumatology identifies two criteria for the diagnosis of fibromyalgia: (1) pain is experienced in 11 of the 18 tender points on palpation and (2) the patient has a history of widespread pain for at least 3 months. The other findings may also be present but are not diagnostic for fibromyalgia.

35. One important nursing intervention for the patient with fibromyalgia is to teach the patient to do what? a. Rest the muscles as much as possible to avoid triggering pain. b. Plan nighttime sleep and naps to obtain 12 to 14 hours of sleep a day. c. Try the use of food supplements such as glucosamine and chondroitin for relief of pain. d. Use stress management techniques such as biofeedback, meditation, or cognitive behavioral therapy

35. d. The pain and related symptoms of fibromyalgia cause significant stress and anxiety is a common finding. Stress management is an important part of the treatment and may include any of the commonly used relaxation strategies as well as psychologic counseling

36. A patient with debilitating fatigue has been diagnosed with chronic fatigue syndrome. Which criteria are considered the four major criteria that must be present for this diagnosis to be made (select all that apply)? a. Unexplained muscle pain b. Fatigue not due to ongoing exertion c. Tender cervical or axillary lymph nodes d. Fatigue not substantially alleviated by rest e. Headaches of a new type, pattern, or severity f. Unexplained, persistent, or relapsing chronic fatigue of new and definite onset g. Fatigue resulting in substantial reduction in occupational, educational, social, or personal activities

36. b, d, f, g. The Centers for Disease Control and Prevention, National Institutes of Health, and International Chronic Fatigue Syndrome Study Group identified the four major criteria for diagnosis of chronic fatigue syndrome: (1) fatigue not due to ongoing exertion; (2) fatigue not substantially alleviated by rest; (3) unexplained, persistent, or relapsing chronic fatigue of new and definite onset; and (4) a reduction in occupational, education, social, or personal activities from fatigue. The other options are some of the minor criteria; four or more minor criteria must be present with the major criteria for 6 months or more before the diagnosis can be made. (See Table 65-20.)

During muscle strength testing the patient has active movement against gravity and some resistance to pressure. What's score should the nurse give this finding?

4

4. To preserve function and the ability to perform activities of daily living (ADLs), what should the nurse teach the patient with OA? a. Avoid exercise that involves the affected joints. b. Plan and organize task performance to be less stressful to joints. c. Maintain normal activities during an acute episode to prevent loss of function. d. Use mild analgesics to control symptoms when performing tasks that cause pain.

4. b. Principles of joint protection and energy conservation are critical in being able to maintain functional mobility in the patient with OA and patients should be helped to find ways to perform activities and tasks with less stress. Range-of motion (ROM), isotonic, and isometric exercises of the affected joints should be balanced with joint rest and protection but during an acute flare of joint inflammation, the joints should be rested. If a joint is painful, it should be used only to the point of pain and masking the pain with analgesics may lead to greater joint injury.

5. A patient with OA asks the nurse whether he could try glucosamine and chondroitin for control of his symptoms. The best response by the nurse includes what information? a. Some patients find these supplements helpful for relieving arthritis pain and improving mobility. b. Although these substances may not help, there is no evidence that they can cause any untoward effects. c. These supplements are a fad that has not been shown to reduce pain or increase joint mobility in patients with OA. d. Only dosages of these supplements available by prescription are high enough to provide any benefit in treatment of OA.

5. a. Some relief for moderate to severe arthritic pain but not for mild arthritic pain has been observed with the use of over-the-counter glucosamine and chondroitin sulfate. These substances should be discontinued if there are no effects after consistent use over 90 to 120 days. They may decrease the effectiveness of antidiabetic drugs and increase the risk of bleeding.

6. A patient taking ibuprofen (Motrin) for treatment of OA has good pain relief but is experiencing increased dyspepsia and nausea with the drug's use. The nurse consults the patient's primary care provider about doing what? a. Adding misoprostol (Cytotec) to the patient's drug regimen b. Substituting naproxen (Naprosyn) for the ibuprofen (Motrin) c. Returning to the use of acetaminophen but at a dose of 5 g/day instead of 4 g/day d. Administering the ibuprofen with antacids to decrease the gastrointestinal (GI) irritation

6. a. Common side effects of nonsteroidal antiinflammatory drugs (NSAIDs) include gastrointestinal (GI) irritation and bleeding, dizziness, rash, headache, and tinnitus. Misoprostol (Cytotec) is used to prevent NSAID-induced gastric ulcers and gastritis and would increase the patient's tolerance of any of the NSAIDs. The use of naproxen would cause the same gastric effects as ibuprofen. The daily dose of acetaminophen should not exceed 4 g/day to prevent liver damage and antacids interfere with the absorption of NSAIDs.

7. Which description is most characteristic of osteoarthritis (OA) when compared to rheumatoid arthritis (RA)? a. Not systemic or symmetric b. Rheumatoid factor (RF) positive c. Most commonly occurs in women d. Morning joint stiffness lasts one to several hours

7. a. OA is not systemic or symmetric. Morning joint stiffness resolves in about 30 minutes. Rheumatoid arthritis (RA) is rheumatoid factor (RF) positive and characterized by being systemic and affecting small joints symmetrically. Morning joint stiffness lasts 60 minutes to all day.

8. What best describes the manifestations of OA? a. Smaller joints are typically affected first. b. There is joint stiffness after periods of inactivity. c. Joint stiffness is accompanied by fatigue, anorexia, and weight loss. d. Pain and immobility may be aggravated by falling barometric pressure.

8. d. Pain and immobility of OA may be aggravated by falling barometric pressure. OA affects weight-bearing joints of knees and hips. Stiffness occurs on arising but usually subsides after 30 minutes. Pain during the day is relieved with rest. Fatigue, anorexia, and weight loss are nonspecific manifestations of the onset of RA.

9. During the physical assessment of the patient with early to moderate RA, what should the nurse expect to find? a. Hepatomegaly b. Heberden's nodes c. Spindle-shaped fingers d. Crepitus on joint movement

9. c. In early disease, the fingers of the patient with moderate RA (1) may become spindle shaped from synovial hypertrophy and thickening of the joint capsule, (2) have no joint deformities but may have limited joint mobility, (3) have adjacent muscle atrophy, and (4) may be inflammed. Splenomegaly may be found with Felty syndrome in patients with severe nodule-forming RA. Heberden's nodes and crepitus on movement are associated with osteoarthritis.

*A nurse is admitting a client to the orthopedic unit following a total knee arthroplasty. Which of the following actions by the nurse are appropriate?* Select all that apply A. Maintain continuous passive motion device. B. Palpate dorsal pedal pulses. C. Place pillow behind the knee. D. Elevate heels off bed. E. Apply heat therapy to incision.

A. CORRECT: A continuous passive motion device promotes motion in the knee and prevents scar tissue formation. B. CORRECT: The nurse should assess the strength of the pulses of both lower extremities to help determine adequate circulation. C. INCORRECT: A pillow should not be placed behind the knee to avoid flexion contractures. D. CORRECT: The nurse should prevent pressure ulcers on the client's heels by elevating the heels off the bed with a pillow. E. INCORRECT: The nurse should apply cold therapy, not heat therapy, to reduce postoperative swelling.

*A nurse is reviewing the plan of care for a client who has systemic lupus erythematosus (SLE). The client reports fatigue, joint tenderness, swelling, and difficulty urinating. Which of the following laboratory findings should the nurse anticipate?* Select all that apply A. Positive ANA tier B. Increased hemoglobin C. 2+ urine protein D. Increased serum C3 and C4 E. Elevated BUN

A. CORRECT: A positive antinuclear antibody (ANA) titer is an expected finding in a client who has SLE. The ANA test identifies the presence of antibody produced against the client's own DNA. B. INCORRECT: Increased hemoglobin is not an expected finding in a client who has SLE. C. CORRECT: Increased urine protein is an expected finding due to renal involvement as a result of SLE. D. INCORRECT: Increased serum C3 and C4 are not expected findings in a client who has SLE. Findings would be decreased. E. CORRECT: Elevated BUN is an expected finding due to renal involvement in a client who has SLE.

*A nurse at a provider's office is caring for an older adult client who is having an annual physical exam. Which of the following findings indicates additional follow-up is needed in regard to the prostate gland?* Select all that apply A. Prostate-specific antigen (PSA) is 7.1 ng/mL. B. A digital rectal exam (DRE) reveals an enlarged and nodular prostate. C. The client reports a weak urine stream. D. The client reports urinating once during the night. E. Smegma is present below the glands of the penis.

A. CORRECT: Although the PSA level typically is elevated in an older adult male, a PSA level of greater than 4 ng/mL warrants additional follow-up. B. INCORRECT: A prostate that is enlarged and smooth is an expected finding in an older adult male. C. CORRECT: A weak urine stream is a clinical manifestation of benign prostatic hyperplasia and warrants follow-up. D. INCORRECT: Urinating once during the night is an expected finding for an older adult male. E. INCORRECT: Smegma is a normal secretion that can accumulate beneath the glans penis.

*A nurse is planning care for a client who is postoperative following an arthroscopy of the knee. Which of the following actions should be taken by the nurse?* Select all that apply A. Assess color and temperature of the extremity. B. Apply warm compresses to incision sites. C. Place pillows under the extremity. D. Administer analgesic medication. E. Assess pulse and sensation in the foot.

A. CORRECT: Assessing color and temperature of the affected extremity identifies alterations in circulation. B. INCORRECT: Cold compresses are applied the first 24 hr to the incisional site to decrease swelling and pain. C. CORRECT: Elevating the leg will help decrease swelling and pain in the affected extremity. D. CORRECT: Administering prescribed opioid medication will help to relieve joint pain in the affected extremity. E. CORRECT: Assessing pulse and sensation of the affected extremity identifies alterations in circulation.

*A nurse is teaching a client how to manage an external fixation device upon discharge. Which of the following statements by the client indicates an understanding?* Select all that apply A. "I will clean the pins more often if drainage from the pins increases." B. "I will use a separate cotton swab for each pin." C. "I will report loosening of the pins to my doctor." D. "I will move my leg by lifting the device in the middle." E. "I will report increased redness at the pin sites."

A. CORRECT: Clean the external fixation pins one to two times each day to remove exudate that may harbor bacteria. B. CORRECT: Using a separate cotton swab on each pin will decrease the risk of cross-contamination, which could cause pin site infection. C. CORRECT: Notify the provider if a pin is loose because the provider will know how much to tighten the pin and prevent damage to the tissue and bone. D. INCORRECT: The external fixation device should never be used to lift or move the affected leg, due to the risk of injuring and dislocating the fractured bone. E. CORRECT: Crusting at the pin site provides a natural barrier from bacteria and should not be removed. NCLEX® Connection:

*A nurse in a provider's office is providing information to a client who has dysfunctional uterine bleeding (DUB). Which of the following statements by the client indicate understanding of the information?* Select all that apply A. "My heavy bleeding may be due to a hormonal imbalance." B. "If I experience menstrual pain, I should take aspirin." C. "Oral contraceptives are contraindicated for women who have heavy uterine bleeding like mine." D. "My doctor may perform a D&C to find out what's causing my abnormal bleeding." E. "My condition is more common in women who are in their 30s."

A. CORRECT: DUB can be caused by a progesterone deficiency. B. INCORRECT: Anovulation is associated with a deficiency in estrogen and progesterone, which contributes to DUB. C. INCORRECT: DUB occurs when progesterone levels are low. Contraceptives that contain progestin can be used to treat the condition. D. CORRECT: When the provider performs a dilatation and curettage, endometrium scraped from the uterine wall is sent to the laboratory for evaluation. E. INCORRECT: DUB is more common in young women who are just starting to menstruate and in women who are nearing menopause.

*A nurse is caring for a client following a below-the-elbow amputation. Which of the following actions should the nurse take?* Select all that apply A. Encourage dependent positioning of the residual limb. B. Inspect for presence and amount of drainage. C. Implement shrinkage intervention of the residual limb. D. Wrap the residual limb in a circular manner using gauze. E. Assess for feelings of body image changes.

A. CORRECT: Dependent positioning of the residual limb can improve circulation to the end of the stump and promote healing. B. CORRECT: Inspecting for the presence and amount of drainage can assist in determining early manifestations of infection. C. CORRECT: Residual limb preparation includes shrinkage interventions before fitting of the prosthesis. D. INCORRECT: Wrap the residual limb with an elastic bandage in a figure-eight manner to prevent restriction of blood flow before fitting for the prosthesis. E. CORRECT: The client may have feelings of depression, anger, withdrawal, and grief due to body image changes, and the nurse should assess for these feelings.

*A nurse is reviewing the medical record of a client who has premenstrual syndrome (PMS). The nurse should identify that which of the following medications are used to treat premenstrual syndrome?* Select all that apply A. Fluoxetine (Prozac) B. Spironolactone (Aldactone) C. Ethinyl estradiol/drospirenone (Yasmin) D. Ferrous sulfate (Feosol) E. Methylergonovine (Methergine)

A. CORRECT: Fluoxetine, an SSRI, is used to treat the emotional symptoms of PMS, such as irritability and mood swings, and has an added effect of treating physical symptoms. B. CORRECT: Spironolactone is a diuretic and can reduce bloating and weight gain associated with PMS. C. CORRECT: Oral contraceptives that contain drospirenone reduce the symptoms of PMS. D. INCORRECT: Oral iron supplements, such as ferrous sulfate, are used to treat anemia associated with dysfunctional uterine bleeding. E. INCORRECT: Methylergonovine is used to treat postpartum hemorrhage.

*A nurse is assessing a client who has osteoarthritis of the knees and fingers. Which of the following manifestations should the nurse expect to find?* Select all that apply A. Heberden's nodes B. Swelling of all joints C. Small body frame D. Enlarged joint size E. Limp when walking

A. CORRECT: Heberden's nodes are enlarged nodules on the distal interphalangeal joints of the hands and feet of a client who has osteoarthritis. B. INCORRECT: Swelling and pain of all joints is a manifestation of rheumatoid arthritis. A local inflammation of a joint is related to osteoarthritis. C. INCORRECT: A small body frame is a risk factor for rheumatoid arthritis. Obesity is a risk factor for osteoarthritis. D. CORRECT: A client may manifest enlarged joints due to bone hypertrophy. E. CORRECT: A client may manifest a limp when walking due to pain from inflammation in the localized joint.

*A nurse is assessing a client who had an external fixation device applied 2 hr ago for a fracture of the left tibia and fibula. Which of the following findings is a manifestation of compartment syndrome?* Select all that apply A. Intense pain when the left foot is passively moved B. Capillary refill of 3 sec on the client's left toes C. Hard, swollen muscle in the client's left leg D. Burning and tingling of the client's left foot E. Client report of minimal pain relief following a second dose of opioid medication

A. CORRECT: Intense pain of the left foot when passively moved may indicate pressure from edema on nerve endings and is a neurological sign of compartment syndrome. B. INCORRECT: Edema of the left toes is an expected finding in a client who has a fracture of the left tibia and fibula. C. CORRECT: A hard, swollen muscle on the affected extremity indicates edema build-up in the area of injury and is a sign of compartment syndrome. D. CORRECT: Burning and tingling of the left foot indicates pressure from edema on nerve endings and is an early neurological sign of compartment syndrome. E. CORRECT: Minimal pain relief after receiving opioid medication may indicate pressure from edema on nerve endings and is an early neurological sign of compartment syndrome.

*A nurse is presenting information to clients at a health fair on measures to reduce the risk of amputation. Which of the follow information should the nurse provide?* Select all that apply A. Encourage clients who smoke to consider smoking cessation programs. B. Encourage clients who have diabetes mellitus to maintain blood glucose within the reference range. C. Instruct clients to unplug electrical equipment when performing repairs. D. Encourage clients who have vascular disease to maintain good foot care. E. Advise clients to wait 2 hr after taking pain medication before driving.

A. CORRECT: Smoking cessation can decrease the development of arteriosclerosis and possible amputation of a lower extremity. B. CORRECT: Regulation of blood glucose within a normal reference range may decrease the development of arteriosclerosis and possible amputation of a lower extremity. C. CORRECT: Unplugging electrical equipment when performing repairs prevents electrocution and injury to an extremity, which may lead to amputation. D. CORRECT: Maintaining good foot care may prevent a possible infection which can result in amputation. E. INCORRECT: Driving under the influence of pain medication may lead to an accident and injury to an extremity requiring amputation.

*A nurse is teaching a client who has a new diagnosis of rheumatoid arthritis (RA). Which of the following statements should the nurse include in the teaching?* A. "You can experience morning stiffness when you get out bed." B. "You can experience abdominal pain." C. "You can experience weight gain." D. "You can experience low blood sugar."

A. CORRECT: The client can experience stiffness in her joints upon rising. This is an appropriate statement for the nurse to give. B. INCORRECT: This is not an appropriate statement for the nurse to give. The client who has RA may experience pleuritic pain (upon inspiration). C. INCORRECT: This is not an appropriate statement for the nurse to give. The client who has RA may experience weight loss. D. INCORRECT: This is not an appropriate statement for the nurse to give. The client who has RA does not experience a low blood sugar.

*A nurse is assessing an older adult client who has arteriosclerosis and is scheduled for a possible right lower extremity amputation. Which of the following are expected findings in the affected extremity?* Select all that apply A. Skin cool to touch from mid-calf to the toes B. Increased sensitivity to fine touch C. Palpable pounding pedal pulse D. Lack of hair on lower leg E. Blackened areas on several toes

A. CORRECT: The client may have coolness of the affected extremity where decreased vascularization starts. B. INCORRECT: The client's affected extremity may become dusky when sitting due to decreased vascularization of the extremity. C. INCORRECT: The client will have a lack of or diminished pedal pulse of the affected extremity due to decreased vascularization. D. CORRECT: The client may have decreased hair growth on areas of the affected extremity due to decreased vascularization. E. CORRECT: The client may have blackened areas on several toes suggestive of gangrene due to decreased vascularization to the affected extremity.

*A nurse is completing a preoperative teaching plan for a client who is to have a total hip arthroplasty. Which of the following should the nurse include in the teaching plan?* Select all that apply A. Encouraging complete autologous blood donation B. Sitting in a low reclining chair C. Having the client roll onto the operative hip D. Using an abductor pillow when turning E. Performing isometric exercises

A. CORRECT: The client should be encouraged to donate blood that can be used postoperatively. B. INCORRECT: The client should anticipate sitting in a high chair to keep the hip at a 90° angle. This prevents dislocation. C. INCORRECT: The client should not turn to the operative side. D. CORRECT: The client should use an abductor device or pillow between the legs when turning. This helps avoid dislocation of the affected hip. E. CORRECT: The client should perform isometric exercises to prevent blood clots and maintain muscle tone.

*A nurse is instructing a client how to perform Kegel exercises. Which of the following instructions should the nurse include?* Select all that apply A. Perform exercises about 50 times a daily. B. Contract the circumvaginal and/or perirectal muscles. C. Gradually increase the contraction period to 10 seconds. D. Follow each contraction with at least a 10-second relaxation period. E. Perform while sitting, lying, and standing. F. Tighten abdominal muscles during contractions.

A. CORRECT: The client should perform a set of exercises at least four times a day. B. CORRECT: The client should contract her circumvaginal and perirectal muscles as if trying to stop the flow of urine or passing flatus. C. CORRECT: The client should hold the contraction for 10 seconds. She might need to gradually increase the contraction period to reach this goal. D. CORRECT: The client should follow each contraction with a period of relaxation of 10 to 15 seconds. E. CORRECT: The client should perform the exercises in all three positions. F. INCORRECT: The client should relax her other muscles, such as those in her abdomen and her thighs.

*A nurse is planning care for a client who will undergo an electromyography (EMG). Which of the following should the nurse include?* Select all that apply A. Assess for bruising. B. Administer aspirin prior to the procedure. C. Determine whether the client takes a muscle relaxant. D. Instruct the client to flex muscles during needle insertion. E. Expect swelling, redness, and tenderness at the insertion sites.

A. CORRECT: The nurse should inform the client that some bruising might occur at the needle insertion sites. B. INCORRECT: The nurse should apply ice to the insertion sites to prevent hematoma from developing. C. CORRECT: The nurse should assess the client's prescribed medications to determine whether the client takes a muscle relaxant, which may create a false reading. D. CORRECT: The nurse should ask the client to flex muscles for an easier insertion of the needle into the muscle. E. INCORRECT: The nurse should instruct the client to report swelling, redness, and tenderness at the insertion sites to the provider because this may indicate an infection.

*A nurse is instructing a client who is scheduled for a transurethral resection of the prostate (TURP) about postoperative care. Which of the following information should the nurse include in the teaching?* A. "You might have a continuous sensation of needing to void even though you have a catheter." B. "You will be on bed rest for the first 2 days after the procedure." C. "You will be instructed to limit your fluid intake after the procedure." D. "Your urine should be clear yellow the evening after the surgery."

A. CORRECT: To reduce the risk of postoperative bleeding, the client will have a catheter with a large balloon that places pressure on the internal sphincter of the bladder. Pressure on the sphincter causes a continuous sensation of needing to void. B. INCORRECT: The client is ambulated early in the postoperative period to reduce the risk of deep‑vein thrombosis and other complications that occur due to immobility. C. INCORRECT: The client is encouraged to increase his fluid intake unless contraindicated by another condition. A liberal fluid intake reduces the risks of urinary tract infection and dysuria. D. INCORRECT: The client's urine is expected to be pink the first 24 hr after surgery.

*A nurse is completing discharge teaching to a client who had a wound debridement for osteomyelitis. Which of the following information should the nurse include?* A. Antibiotic therapy should continue for 3 months. B. Relief of pain indicates the infection is eradicated. C. Airborne precautions are used during wound care. D. Expect paresthesia distal to the wound.

A. CORRECT: Treatment of osteomyelitis includes continuing antibiotic therapy for 3 months. B. INCORRECT: Relief of pain does not indicate that osteomyelitis is resolved, and the client should continue antibiotic therapy as prescribed. C. INCORRECT: When performing wound care, standard precautions are implemented, because a wound due to osteomyelitis is classified as a dirty wound and is usually left open for healing. D. INCORRECT: Clean technique is used when performing a dressing change of a wound due to osteomyelitis, because the wound is classified as a dirty wound and is usually left open for healing.

*A nurse is planning discharge teaching for a client who had a total hip arthroplasty. Which of the following should the nurse include in the teaching?* Select all that apply A. Clean the incision daily with soap and water. B. Turn the toes inward when sitting or lying. C. Sit in a straight-backed armchair. D. Bend at the waist when putting on socks. E. Use a raised toilet seat.

A. CORRECT: Washing the surgical incision daily with soap and water decreases the risk of infection. B. INCORRECT: Toes should be externally rotated. This prevents dislocation of the hip prosthesis. C. CORRECT: The client who uses a straight-backed armchair decreases the chance of bending at a greater than 90° angle, which may cause dislocation of the hip prosthesis. D. INCORRECT: The client who bends at the waist places the hip in a position greater than a 90° angle, which may cause dislocation of the hip prosthesis. E. CORRECT: The client who uses a toilet riser decreases the chance of bending greater than 90° degrees, which may cause dislocation of the hip prosthesis.

When obtaining information about the patients use of medications, the nurse recognizes that both bone and muscle function may be impaired when the patient reports taking what type of drug? A. Corticosteroids B. Oral hypoglycemic agents C. Potassium-depleting diuretics D. Nonsteroidal antiinflammatory drugs (NSAIDs)

A. Corticosteroids

*A nurse is providing information to a client who is scheduled for a transrectal ultrasound (TRUS). Which of the following information should the nurse include?* A. "This procedure will determine whether you have prostate cancer." B. "The procedure is contraindicated if you have an allergy to eggs." C. "Sound waves will be used to create a picture of your prostate." D. "You should avoid having a bowel movement for 1 hr prior to the procedure."

A. INCORRECT: A biopsy is used to make the diagnosis of prostate cancer. B. INCORRECT: A rectal probe transducer is inserted into the client's rectum when a TRUS is performed. C. CORRECT: A TRUS creates an image of the prostate using sound waves. D. INCORRECT: Anesthesia is not used for this procedure.

*A nurse is completing preoperative teaching for a client who is to undergo an arthroscopy to repair an injury to a shoulder. Which of the following statements should the nurse include?* Select all that apply A. "Avoid damage or moisture to the cast on your arm." B. "Inspect your incision daily for indications of infection." C. "Apply ice packs to the area for the first 24 hours." D. "Position your arm in a dependent position." E. "Perform isometric exercises."

A. INCORRECT: A sling is applied to immobilize the arm of the affected shoulder to limit activity and promote healing. B. CORRECT: The incision should be inspected daily for evidence of infection, such as redness, swelling, or purulent drainage. C. CORRECT: Normal activity will resume as prescribed by the provider. D. INCORRECT: The affected extremity should be placed across the chest in a nondependent position to reduce swelling. E. CORRECT: Isometric exercises are recommended by the provider and directed by the physical therapist.

*A nurse is caring for a client who has rheumatoid arthritis. Which of the following laboratory tests are used to diagnose this disease?* Select all that apply A. Urinalysis B. Erythrocyte sedimentation rate (ESR) C. BUN D. Antinuclear antibody (ANA) titer E. WBC count

A. INCORRECT: A urinalysis is not a laboratory test used to diagnose rheumatoid arthritis. This test can used for detecting kidney failure. B. CORRECT: ESR is a laboratory test used to diagnose rheumatoid arthritis. This laboratory test will show an elevated result in clients who have rheumatoid arthritis. C. INCORRECT: A BUN is not a laboratory test used to diagnose rheumatoid arthritis. This test can be used for detecting kidney failure. D. CORRECT: ANA titer is a laboratory test used to diagnose rheumatoid arthritis. This laboratory test will show a positive result in clients who have rheumatoid arthritis. E. CORRECT: WBC count is a laboratory test used to diagnose rheumatoid arthritis. This laboratory test will show an elevated result in clients who have rheumatoid arthritis.

*A nurse is reviewing the health record of a client who is to undergo total joint arthroplasty. The nurse should recognize which of the following findings as a contraindication to this procedure?* A. Age of 78 B. History of cancer C. Previous joint replacement D. Bronchitis 2 weeks ago

A. INCORRECT: Age greater than 70 is not a contraindication for a total joint arthroplasty unless there are comorbidity factors. B. INCORRECT: History of cancer is not a contraindication for a total joint arthroplasty unless there are comorbidity factors. C. INCORRECT: Previous joint arthroplasty surgery is not contraindicated for total joint arthroplasty unless there are comorbidity factors. D. CORRECT: A recent infection can cause micro-organisms to migrate to the surgical area and cause the prosthesis to fail.

*A school nurse is providing an education session about menstruation with a group of adolescent students. Which of the following statements should the nurse include?* Select all that apply A. "The average age of onset of menstruation is 10." B. "The range for a typical menstrual cycle is between 23 and 25 days." C. "The first day of the menstrual cycle begins with the last day of the menstrual period." D. "Ovulation typically occurs around the 14th day of the menstrual cycle." E. "A menstrual period can last as long as 9 days."

A. INCORRECT: Although some females experience the onset of menstruation as early as age 11, the average age is 13. B. CORRECT: Although a typical menstrual cycle is 28 days, a range of 21 to 42 days is considered a regular menstrual cycle. C. INCORRECT: The first day of the menstrual cycle begins with the first day of the menstrual period. D. CORRECT: The first half of the menstrual cycle is the follicular phase, and the second half of the menstrual cycle is the luteal phase. Ovulation typically occurs around the middle of the cycle, or day 14 in a 28-day menstrual cycle. E. CORRECT: A menstrual period typically lasts from 4 to 7 days.

*A nurse in a provider's office is obtaining a history from a client who is undergoing an evaluation for benign prostatic hyperplasia (BPH). The nurse should identify that which of the following findings are indicative of this condition?* Select all that apply A. Backache B. Frequent urinary tract infections C. Weight loss D. Hematuria E. Urinary incontinence

A. INCORRECT: Backache occurs in the presence of prostate cancer that has spread to other areas of the body. B. CORRECT: In the presence of BPH, pressure on urinary structures leads to urinary stasis, which in turn promotes the occurrence of urinary tract infections. C. INCORRECT: Weight loss occurs in the presence of prostate cancer. D. CORRECT: Painless hematuria occurs in the presence of BPH. E. CORRECT: Overflow incontinence occurs in the presence of BPH due to an increased volume of residual urine.

*A nurse is educating clients at a health fair about dual x-ray absorptiometry (DXA) scans. Which of the following should the nurse include?* Select all that apply A. The test requires the use of contrast material. B. The hip and spine are the usual areas the device scans. C. The scan is used to detect osteoarthritis. D. Bone pain may indicate a need for a scan. E. Females should have a baseline scan during their 40s.

A. INCORRECT: Contrast material is not used for a DXA scan. B. CORRECT: The most common areas for a DXA scan is the hip and spine for more clear visualization of a large area of bone. C. INCORRECT: Osteoporosis, not osteoarthritis, is detected by a DXA scan. D. CORRECT: Bone pain, loss of height, and fractures are findings that may indicate the need for a DXA scan. E. CORRECT: A baseline scan at age 40 is recommended to compare with a scan done during the postmenopausal period.

*A client had an above-the-knee amputation. The client reports a sharp, stabbing type of phantom pain. Which of the following actions should the nurse take?* A. Remove the initial pressure dressing. B. Encourage use of cold therapy. C. Question whether the pain is real. D. Administer an antiepileptic medication.

A. INCORRECT: Counseling services can assist clients to cope with body image changes and is not prescribed for treatment of phantom pain. B. INCORRECT: Heat therapy, not cold therapy, to the residual limb is an alternative therapy that may relieve phantom pain. C. INCORRECT: Phantom pain is related to the severed nerve pathways following the amputation, and the nurse should not question whether the pain is real. D. CORRECT: Antiepileptic medication may relieve the client's phantom pain.

*A nurse is providing support to a client who has a new diagnosis of endometriosis. The nurse should inform the client that which of the following conditions is a possible complication of endometriosis?* A. Insulin resistance B. Infertility C. Vaginitis D. Pelvic inflammatory disease

A. INCORRECT: Insulin resistance is a complication of polycystic ovary syndrome. B. CORRECT: Infertility is a complication of endometriosis because endometrial tissue overgrowth can block the fallopian tubes. C. INCORRECT: Vaginitis is typically caused by an infection. D. INCORRECT: Pelvic inflammatory disease is caused by an infection of the pelvic organs.

*A nurse is caring for a client who has a new diagnosis of benign prostatic hyperplasia (BPH). The nurse should expect a prescription for which of the following medications?* A. Oxybutynin (Ditropan) B. Diphenhydramine C. Ipratropium (Atrovent) D. Tamsulosin (Flomax)

A. INCORRECT: Oxybutynin is an anticholinergic medication that is used to treat overactive bladder. Anticholinergic medications are contraindicated for a client who has BPH. Oxybutynin causes urinary retention. B. INCORRECT: Diphenhydramine is an antihistamine and is contraindicated for a client who has BPH. Diphenhydramine causes urinary retention. C. INCORRECT: Ipratropium is an anticholinergic used to treat asthma and other respiratory conditions. Anticholinergic medications are contraindicated for a client who has BPH. Ipratropium causes urinary retention. D. CORRECT: Tamsulosin is an alpha-adrenergic receptor antagonist that relaxes the bladder outlet and the prostate gland, which improves urinary flow.

*A nurse is performing a preoperative assessment for a client who is scheduled for an anterior colporrhaphy. Which of the following client statements should the nurse expect as an indication for this procedure?* A. "I have to push the feces out of a pouch in my vagina with my fingers." B. "I have pain and bleeding when I have a bowel movement." C. "I have had frequent urinary tract infections." D. "I am embarrassed by uncontrollable flatus."

A. INCORRECT: Pouching of feces is a physiological alteration associated with a rectocele. The surgical procedure for a rectocele is posterior colporrhaphy. B. INCORRECT: Pain and bleeding with a bowel movement is a physiological alteration associated with a rectocele. The surgery for a rectocele is a posterior colporrhaphy. C. CORRECT: Due to urinary stasis associated with a cystocele, this finding is consistent with a cystocele. The surgery for a cystocele is an anterior colporrhaphy. D. INCORRECT: Uncontrollable flatus is a physiological alteration associated with a rectocele. The surgery for a rectocele is a posterior colporrhaphy.

*A nurse in the emergency department is planning care for a client who has a right hip fracture. Which of the following immobilization devices should the nurse anticipate in the plan of care?* A. Skeletal traction B. Buck's traction C. Halo traction D. Bryant's traction

A. INCORRECT: Skeletal traction is an immobilization device applied surgically to a long bone fracture. B. CORRECT: Buck's traction is a temporary immobilization device applied to diminish muscle spasms and immobilize the affected extremity until surgery is performed. C. INCORRECT: Halo traction immobilizes the cervical spine when a cervical fracture occurs. D. INCORRECT: Gardner-Wells traction uses tongs to immobilize and realign the cervical spine when a cervical fracture occurs.

*A nurse is assessing a client who is to undergo a right knee arthroplasty. The nurse should expect which of the following findings?* Select all that apply A. Skin reddened over the joint B. Pain when bearing weight C. Joint crepitus D. Swelling of the affected joint E. Limited joint motion

A. INCORRECT: Skin over the knee that's red may indicate infection and is not an expected finding. B. CORRECT: Pain when bearing weight due to degeneration of the joint tissue is an expected finding. C. CORRECT: Joint crepitus due to degeneration of the joint tissue is an expected finding. D. CORRECT: Swelling of the affected joint due to degeneration of the joint tissue is an expected finding. E. CORRECT: Limited joint motion is due to degeneration of the joint tissue and is an expected finding.

*A nurse is caring for a client who has SLE and is experiencing an episode of Raynaud's phenomenon. Which of the following findings should the nurse anticipate?* A. Swelling of joints of the fingers B. Pallor of toes with cold exposure C. Feet become reddened with ambulation D. Client report of intense feeling of heat in the fingers

A. INCORRECT: Swelling, pain, and joint tenderness are clinical findings in a client who has SLE and is not specific to an episode of Raynaud's phenomenon. B. CORRECT: Pallor of the extremities occurs in Raynaud's phenomenon in a client who has SLE and has been exposed to cold or stress. C. INCORRECT: The extremities becoming red, white, and blue when exposed to cold or stress is characteristic of an episode of Raynaud's phenomenon in a client who has SLE. D. INCORRECT: A client report of intense pain in the hands and feet is characteristic of an episode of Raynaud's phenomenon in a client who has SLE.

*A nurse is providing instructions to a client prior to an initial mammogram. Which of the following information should the nurse provide prior to the procedure?* A. "You should not take any aspirin products prior to the mammogram." B. "Do not apply any deodorant the day of the exam." C. "You will need to avoid sexual intercourse the day before the mammogram." D. "You should avoid exercise prior to the exam."

A. INCORRECT: Taking a multivitamin does not alter the accuracy of a mammogram. B. CORRECT: Applying deodorant or powder can alter the accuracy of a mammogram by causing a shadow to appear. C. INCORRECT: Having sexual intercourse does not alter the accuracy of a mammogram. D. INCORRECT: Exercising does not alter the accuracy of a mammogram.

*A nurse in a clinic is reviewing the facility's testing process and procedures for human immune deficiency virus (HIV) with a new employee. Which of the following information should the nurse include?* A. In the presence of HIV, the enzyme immunoassay (EIA) test is typically reactive within 72 hr after the client is infected. B. The Western blot assay is used to confirm the diagnosis of HIV. C. The polymerase chain reaction (PRC) test is used to confirm the diagnosis of HIV. D. CD4+ cell counts will be elevated in a client who is infected with HIV.

A. INCORRECT: The EIA test is typically reactive 3 weeks to 3 months after the infection occurs, but it can be delayed for as long as 36 months. B. CORRECT: Confirming HIV is a two-step process. If the EIA is positive, a second test, the Western blot assay, is done. C. INCORRECT: The PRC test is used to confirm the diagnosis of genital herpes. D. INCORRECT: The EIA test is typically reactive 3 weeks to 3 months after the infection occurs, but it can be delayed for as long as 36 months.

*A nurse is preparing to discharge a client who has had an anterior and posterior colporrhaphy. Which of the following instructions should the nurse provide?* A. "Do not bend over for at least 6 weeks." B. "You can lift objects as heavy as 10 pounds." C. "Do not engage in intercourse for at least 6 weeks." D. "You may have foul-smelling draining within the first week after surgery."

A. INCORRECT: The client does not have a restriction regarding bending over. B. INCORRECT: The client should not lift an object that weighs more than 5 lb. C. CORRECT: The client should refrain from intercourse to allow time for the surgical site to heal, which is typically about 6 weeks. D. INCORRECT: Foul-smelling draining is a sign of infection, which should be reported to the provider.

*A nurse is preparing a client for her first Papanicolaou (Pap) test. Which of the following statements should the nurse to make?* A. "You should urinate immediately after the procedure is over." B. "You will not feel any discomfort." C. "You may experience some bleeding after the procedure." D. "You will need to hold your breath during the procedure."

A. INCORRECT: The client is instructed to urinate immediately before the procedure. B. INCORRECT: The client can experience discomfort when the provider obtains the cervical sample. C. CORRECT: The client can experience a small amount of vaginal bleeding due to scraping of the cervix. D. INCORRECT: The client should use relaxation techniques such as taking deep breaths during the procedure.

*A nurse is providing discharge instructions to a client who is postoperative following a TURP. Which of the following instructions should the nurse include?* Select all that apply A. Avoid sexual intercourse for 3 months after the surgery. B. If urine appears bloody, stop activity and rest. C. Avoid drinking caffeinated beverages. D. Take a stool softener once a day. E. Treat pain with ibuprofen (Motrin).

A. INCORRECT: The client should follow the provider's instructions, which typically includes avoidance of sexual intercourse for 2 to 6 weeks after the surgery. B. CORRECT: Excessive activity can cause recurrence of bleeding. The client should rest to promote reclotting at the incisional site. C. CORRECT: Caffeine is a bladder stimulant and should be avoided. D. CORRECT: The client should take a stool softener to keep the stool soft and thus prevent the complication of bleeding at the time of a bowel movement. E. INCORRECT: The client should avoid taking nonsteroidal anti-inflammatory medications because they can cause bleeding.

*A nurse is providing teaching to a client who is going to have a bone scan. Which of the following statements should the nurse include?* A. "You will receive an injection of a radioactive isotope when the scanning procedure begins." B. "You will be placed in a tube-like structure during the procedure." C. "You will need to take radioactive precautions with your urine for 24 hours after the procedure." D. "You will have to urinate just before the procedure."

A. INCORRECT: The entire procedure will take several hours. The radionuclide substance is injected and must be absorbed by the bone prior to the procedure. The client must wait several hours after the injection before the scan can be completed. B. INCORRECT: The client is placed in a tubelike structure during an MRI. C. INCORRECT: Radioactive precautions related to the client's urine are not necessary following the procedure. D. CORRECT: A radioactive substance is injected 2 to 3 hr before the procedure to allow for visualization of tumors, fractures, or diseases of the bone.

*A nurse is preparing a plan of care to prevent a client from developing flexion contractions following a below-the-knee amputation 24 hr ago. Which of the following should the nurse include?* A. Limit any type of exercise to the residual limb for the first 48 hr after surgery. B. Position the client prone several times each day. C. Wrap the stump in a figure-eight pattern. D. Encourage sitting in a chair during the day.

A. INCORRECT: To avoid flexion contractures, the residual limb should not be elevated with a pillow after 24 hr of the amputation. B. CORRECT: The client should lie prone several times each day for 20 to 30 min to prevent flexion contractures developing. C. INCORRECT: The client may have the residual limb wrapped in a figure eight to prepare for the prosthesis, but this action does not prevent flexion contractures. D. INCORRECT: The client may develop flexion contractures by allowing the residual stump to hang in a bent position when sitting for an extended period following the amputation.

*A nurse is providing teaching to a client who is to undergo cervical biopsy. Which of the following information should the nurse include?* Select all that apply A. "The procedure is painless." B. "Avoid heavy lifting for approximately 2 weeks after the procedure." C. "Heavy bleeding is common during the first 12 hours after the procedure." D. "Plan to rest for the first 72 hours after the procedure." E. "Avoid the use of tampons for 2 weeks after the procedure."

A. INCORRECT: Typically, the client will experience temporary discomfort and cramping when the specimen is obtained. B. CORRECT: The client should avoid heavy lifting until the cervix has healed, which is approximately 2 weeks. C. INCORRECT: Some bleeding is common after a cervical biopsy, but excessive bleeding is a complication and should be reported to the provider. D. INCORRECT: The client should plan to rest for the first 24 hr after the procedure. E. CORRECT: The client should not use tampons until the cervix has healed, which is approximately 2 weeks.

*A nurse is admitting a client who has a new diagnosis of SLE. Which of the following clinical findings should the nurse suspect?* A. Weight gain B. Petechiae on thighs C. Systolic murmur D. Alopecia

A. INCORRECT: Weight gain may occur in the client who has SLE and is being treated with corticosteroids. B. INCORRECT: A butterfly rash on the face is a clinical finding in a client who has SLE. C. INCORRECT: Alopecia is an expected clinical finding in a client who has SLE D. CORRECT: Areas of hair loss are an expected finding in a client who has SLE.

*A nurse is providing teaching for a client who has a history of lower back injury. Which of the following instructions should the nurse give the client to prevent further problems with low back pain?* Select all that apply A. engage in regular exercise including walking B. sit for up to 10 hr each day to rest the back C. maintain weight within 25% of ideal body weight D. create a smoking cessation plan E. wear low-heeled shoes

A. engage in regular exercise including walking D. create a smoking cessation plan E. wear low-heeled shoes

*A nurse is providing information about capsaicin cream to a client who reports continuous knee pain from osteoarthritis. Which of the following information should the nurse include in the discussion?* A. Continuous pain relief is provided. B. Put on gloves before applying the cream to other parts of the body C. Leave cranes on the hands for 10 min following application. D. Apply the medication every 2 hr during the day.

B. Put on gloves before applying the Crean to other parts of the body

The patient was referred to the office by the school nurse for a lateral curvature of the spine. The nurse knows this is called A. Lordosis B. Scoliosis C. Ankylosis D. Kyphosis

B. Scoliosis

*A nurse is caring for a client who received a lower back injury during a fall and describes sharp pain in the back and down the left leg. In which of the following positions should the nurse plan to place the client to attempt to decrease the pain?* A. prone without use of pillows B. semi-fowlers with a pillow under the knees C. high-fowlers with the knees flat on the bed D. supine with the head flat

B. semi-fowlers with a pillow under the knees

When assessing the patient the nurse notes that the patient has foot drop it in the foot slaps down on the floor as the patient walks. How does the partners document the gait? A. Ataxic gait B. Spastic gait C. Antalgic gait D. Steppage gait

D. Steppage gait

While having his height measured during a routine health examination a 79-year-old man asked the nurse why he is shrinking. How should the nurse explained to decrease tight occurs with aging? A. Decreased muscle mass results in stooped posture B. Loss of cartilage in the knees and hip joints causes a loss of height C. Long bones become less dense and shorten as the bone tissue compacts D. Vertebrae become more compressed with thinning of intervertebral discs

D. Vertebrae become more compressed with thinning of intervertebral discs

What test provides fast, precise measurement of the bone mass of the spine, forearm, and total body to evaluate osteoporosis? A. Bone scan B. Diskogram C. Quantitative ultrasound(QUS) D. Dual energy x-ray absorptiometry

D. dual energy x-ray absorptiometry (DEXA)

*The first nursing intervention for the patient who has been sexually assaulted is to* a. treat urgent medical problems b. contact support person for the patient c. provide supplies for the patient to cleanse self d. document bruises and lacerations of the perineum and the cervix

a

A HCP diagnoses a patient with a plantar wart. What should the nurse know about this kind of abnormality a. Painful papillomatous growth on the sole of the foot b. Thickening of skin on the weight-bearing part of the foot c. Local thickening of skin caused by pressure on bony prominences d. Tumor on nerve tissue between the third and fourth metatarsal heads

a

What should the nurse teach the patient recovering from an episode of acute low back pain a. perform daily exercise as a lifelong routine b. Sit in a chair with the hips higher than the knees c. Avoid occupations in which the use of the body is required d. Sleep on the abdomen or on the back with the legs extended

a

7. What should be included in the management during the first 48 hours after an acute soft tissue injury of the ankle (select all that apply)? a. Use of elastic wrap b. Initial immobilization and rest c. Elevation of ankle above the heart d. Alternating the use of heat and cold e. Administration of antiinflammatory drugs

a, b, c, e. Consider the principle of RICE. Rest: movement should be restricted. Ice: cold should be used to promote vasoconstriction and reduce edema. Compression: helps decrease swelling. Elevate: the extremity above the level of the heart. Mild nonsteroidal antiinflammatory drugs (NSAIDs) may be needed to manage pain. Warm, moist compresses may be used after 48 hours for 20 to 30 minutes at a time to reduce swelling and provide comfort.

9. The x-ray shows that the patient's fracture is at the remodeling stage. What characteristics of the fracture healing process occur at this stage (select all that apply)? a. Radiologic union b. Absorption of excess bone cells c. Return to preinjury strength and shape d. Semisolid blood clot at the ends of fragments e. Deposition and absorption of bone in response to stress f. Unorganized network of bone woven around fracture parts

a, b, c, e. When the remodeling stage of healing occurs, radiologic union is present. Excess bone tissue is resorbed in the final stage of healing and union is complete. The bone gradually returns to its preinjury structure strength and shape. The osteoblasts and osteoclasts function normally in response to physical loading stress. The fracture hematoma stage occurs when the hematoma at the ends of the fragments becomes a semisolid blood clot. There is an unorganized network of bone composed of cartilage, osteoblasts, calcium, and phosphorus woven around fracture parts in the callus formation stage.

Which factors are associated with endometrial cancer (select all that apply)? a. Obesity b. Smoking c. Family history d. Early sexual activity e. Early menarche and late menopause f. Unopposed estrogen-only replacement therapy

a, b, e, f. Endometrial cancer is at higher risk in obese patients because adipose cells store estrogen, which is the major risk factor, especially unopposed estrogen. Smoking is a risk factor for endometrial and cervical cancer. Early sexual activity is a risk factor for cervical cancer. Family history and early menarche and late menopause causing increased menstrual cycles are risk factors for ovarian cancer.

What accurately describes prostate cancer detection and/or treatment (select all that apply)? a. The symptoms of pelvic or perineal pain, fatigue, and malaise may be present. b. Palpation of the prostate reveals hard and asymmetric enlargement with areas of induration or nodules. c. Orchiectomy is a treatment option for all patients with prostatic cancer except those with stage IV tumors. d. The preferred hormonal therapy for treatment of prostate cancer includes estrogen and androgen receptor blockers. e. Early detection of cancer of the prostate is increased with annual rectal examinations and serum prostatic acid phosphatase (PAP) measurements. f. An annual prostate examination is recommended starting at age 45 for African American men because of the increased mortality rate from prostatic cancer in this population.

a, b, f. Pelvic or perineal pain, fatigue, malaise, and a hard asymmetric prostate may be present with prostate cancer. Annual prostate examination is recommended starting at a younger age for African American men because of increased diagnosis and mortality from prostate cancer in this ethnic group. An orchiectomy may be done with prostatectomy or for metastatic stages of prostate cancer. Hormonal treatment includes androgen deprivation therapy, luteinizing hormone-releasing hormone agonists, and androgen receptor blockers. Early detection of prostate Cancer is best detected with annual rectal exams and serum PSA. Elevated prostatic acid phosphatase (PAP) will be seen with metastasis, not a new diagnosis.

In performing range of motion with a patient, the nurse put each joint through its full movement. Which joints are capable of abduction and adduction? a. Hip b. Knee c. Wrist d. Elbow e. Thumb f. Shoulder

a, c, e, f Rationale: Abduction is moving the part away from the midline of the body and adduction is moving the part toward the midline of the body. These movements can be done with the hip, wrist, thumb, and shoulder. The knee and elbow move with flexion and extension.

22. What emergency considerations must be included with facial fractures (select all that apply)? a. Airway patency b. Oral examination c. Cervical spine injury d. Cranial nerve assessment e. Immobilization of the jaw

a, c. Airway patency and cervical spinal cord injury are the emergency considerations with facial fractures. Oral examination and cranial nerve assessment will be done after the patient is stabilized. Immobilization of the jaw is done surgically for a mandibular fracture.

Which manifestations of menopause are related to estrogen deficiency (select all that apply)? a. Cessation of menses b. Breast engorgement c. Vasomotor instability d. Reduction of bone fractures e. Decreased cardiovascular risk

a, c. The lack of estrogen in menopause contributes to many of the signs of aging, including cessation of menses, vasomotor instability (hot flashes), atrophic changes of vaginal and external genitalia and breast tissue, increased risks for osteoporosis and coronary artery disease, redistribution of fat, muscle and joint pain, loss of skin elasticity, and atrophic lower urinary tract changes.

*Which factors would place a patient at higher risk for prostate cancer?* Select all that apply a. older than 65 years b. Asian or Native American c. long-term use of an indwelling urethral catheter d. father diagnosed and treated for early stage prostate cancer e. previous history of undescended testicle and testicular cancer

a, d.

The patient calls the office and says that she thinks she has a "yeast infection." What signs or symptoms should the nurse expect in this patient (select all that apply)? a. Intense itching and dysuria b. Hemorrhagic cervix and vagina c. Pruritic, frothy greenish or gray discharge d. Thick, white, cottage cheese-like discharge e. Mucopurulent discharge and postcoital spotting

a, d. "Yeast infection" or vulvovaginal candidiasis has intense itching and dysuria from urine coming in contact with fissures or irritated areas in the vulva. The discharge is thick, white, and curdlike. Hemorrhagic cervix and vagina occur with trichomoniasis and produce a pruritic, frothy greenish or gray discharge. Mucopurulent discharge and postcoital spotting from cervical inflammation are seen with cervicitis.

*A nurse is performing health screenings of clients at a health fair. Which of the following clients have a risk for osteoporosis?* Select all that apply a. A 40yr old client who has been taking prednisone for 4 months b. A 30yr old client who jogs 3 miles daily c. A 45yr old client who takes phenytoin for seizures d. A 65yr old client who has a sedentary lifestyle e. A 70yr old client who has smoked for 50 years

a. A 40yr old client who takes prednisone for asthma c. A 45yr old client who takes phenytoin for seizures d. A 65yr old client who has a sedentary lifestyle e. A 70yr old client who has smoked for 50 years

What is an appropriate outcome for a patient who undergoes an anterior colporrhaphy? a. Maintain normal bowel patterns b. Adjust to temporary ileal conduit c. Urinate within 8 hours postoperatively d. Experience healing of excoriated vaginal and vulvar tissue

a. An anterior colporrhaphy involves repair of a cystocele and an indwelling urinary catheter is left in place for several days postoperatively while healing occurs. Bowel function should not be altered and is maintained with a low-residue diet and a stool softener if necessary to avoid straining and pressure on the incision.

*Which persons are at high risk for chronic low back pain?* Select all that apply a. A 63-yr-old man who is a long-distance truck driver b. A 30-yr-old nurse who works on an orthopedic unit and smokes c. A 55-yr-old construction worker who is 6 ft, 2 in and weighs 250 lb d. A 44-yr-old female chef with prior compression fracture of the spine e. A 28-yr-old female yoga instructor who is 5 ft, 6 in and weighs 130 lb

a. A 63-yr-old man who is a long-distance truck driver b. A 30-yr-old nurse who works on an orthopedic unit and smokes c. A 55-yr-old construction worker who is 6 ft, 2 in and weighs 250 lb d. A 44-yr-old female chef with prior compression fracture of the spine rationale: Causes of chronic low back pain include (1) degenerative conditions, such as arthritis or disc disease; (2) osteoporosis; (3) weakness from the scar tissue of prior injury; and (4) chronic strain on lower back muscles from obesity, pregnancy, or jobs that require repetitive heavy lifting, vibration, and prolonged sitting. Health care personnel are at high risk for low back pain. Lifting and moving patients, excessive time being stooped over or leaning forward, and frequent twisting can cause low back pain.

26. Which kind of hip fracture is usually repaired with a hip prosthesis? a. Intracapsular b. Extracapsular c. Subtrochanteric d. Intertrochanteric

a. A hip prosthesis is usually used for intracapsular fractures. The other options are all for extracapsular fractures.

11. A patient with a fractured femur develops the complication of malunion. What does the nurse recognize has happened to the patient? a. The fracture heals in an unsatisfactory position. b. The fracture fails to heal properly despite treatment. c. Fracture healing progresses more slowly than expected. d. Loss of bone substances occurs as a result of immobilization.

a. A malunion occurs when the bone heals in the expected time but in an unsatisfactory position, possibly resulting in deformity or dysfunction. Nonunion occurs when the fracture fails to heal completely despite treatment. Delayed union is healing of the fracture at a slower rate than expected. In posttraumatic osteoporosis, the loss of bone substances occurs as a result of immobilization.

What is the function of a tendon? A. Attaches muscle to the bone B. Connects bone to bone at the joints C. Connects cartilage to muscle on joints D. Attaches Synovium to the joint capsule

a. Attaches muscle to bone Rationale: The ligament attaches bone to bone at the joint. Fascia encloses individual muscles but does not connect cartilage to the muscle in joints. The bursae are lined with a synovial membrane and are located in joints to relieve pressure and decrease friction between moving parts.

34. Priority Decision: An immediate postoperative prosthetic fitting during surgery is used for a patient with a traumatic below-the-knee amputation. During the immediate postoperative period, what is a priority nursing intervention? a. Monitor the patient's vital signs. b. Assess the incision for hemorrhage. c. Elevate the residual limb on pillows. d. Have the patient flex and extend the knee every hour.

a. Because the device covers the residual limb, the surgical site cannot be directly seen, and postoperative hemorrhage is not apparent on dressings. This requires vigilant assessment of vital signs for signs of bleeding. Elevation of the residual limb with an immediate prosthetic fitting is not necessary because the device itself prevents edema formation. Leg exercises are not done in the immediate postoperative period to avoid disruption of ligatures and the suture line.

29. A patient with a fractured right hip has an anterior ORIF of the fracture. What should the nurse plan to do postoperatively? a. Get the patient up to the chair on the first postoperative day. b. Ambulate the patient with partial weight bearing by discharge. c. Keep the leg abductor pillow on the patient even when bathing. d. Position the patient only on the back and the nonoperative side.

a. Because the fracture site is internally fixed with pins or plates, the fracture site is stable and the patient is moved from the bed to the chair on the first postoperative day. Ambulation begins on the first or second postoperative day without weight bearing on the affected leg. Weight bearing on the affected extremity is usually restricted for 6 to 12 weeks until adequate healing is evident on x-ray. Abductor pillows are used for patients who have total hip replacements by the posterior surgical approach. The patient may be positioned on the operative side following internal fixation as prescribed by the HCP.

A patient is seeking medical intervention for erectile dysfunction. Why should he be thoroughly evaluated? a. Treatment of erectile dysfunction is based on the cause of the problem. b. Psychologic counseling can reverse the problem in 80% to 90% of the cases. c. New invasive and experimental techniques currently used have unknown risks. d. Most treatments for erectile dysfunction are contraindicated in patients with systemic diseases.

a. Before treatment for erectile dysfunction is initiated, the cause must be determined so that appropriate treatment can be planned. Only a small percentage of erectile dysfunction is caused by psychologic factors. In the case of the 80% to 90% of erectile dysfunction that is of physiologic causes, interventions are directed at correcting or eliminating the cause or restoring function by medical means. New invasive or experimental treatments are not widely used and should be limited to research centers and patients with systemic diseases can be treated medically if the cause cannot be eliminated.

During assessment of the patient with vulvar cancer, what should the nurse expect to find? a. Soreness and itching of the vulva b. Labial lesions with purulent exudate c. Severe excoriation of the labia and perineum d. Painless, firm nodules embedded in the labia

a. Early signs of cancer of the vulva include pruritus, or burning, soreness of the vulva, and discharge or bleeding of the vulva, with edema of the vulva and lymphadenopathy occurring as the disease progresses. Labial lesions and excoriation more commonly occur with infections, and nodules are more often cysts or lipomas.

What is the effect of finasteride (Proscar) in the treatment of BPH? a. A reduction in the size of the prostate gland b. Relaxation of the smooth muscle of the urethra c. Increased bladder tone that promotes bladder emptying d. Relaxation of the bladder detrusor muscle promoting urine flow

a. Finasteride results in suppression of androgen formation by inhibiting the formation of the testosterone metabolite dihydroxytestosterone, the principal prostatic androgen, and results in a decrease in the size of the prostate gland. α-Adrenergic blockers are used to cause smooth muscle relaxation in the prostate that improves urine flow. Drugs affecting bladder tone are not indicated.

35. What is the reason a nurse should position a patient prone several times a day after an above-the-knee amputation with a delayed prosthetic fitting? a. To prevent flexion contractures b. To assess the posterior skin flap c. To reduce edema in the residual limb d. To relieve pressure on the incision site

a. Flexion contractures, especially of the hip, may be debilitating and delay rehabilitation for the patient with a leg amputation. To prevent hip flexion, the patient should avoid sitting in a chair with the hips flexed or having pillows under the surgical extremity for prolonged periods. The patient should lie on the abdomen for 30 minutes 3 to 4 times a day to extend the hip.

A patient is diagnosed and treated for a Gardnerella vaginalis infection at a clinic. For her treatment to be effective, what does the nurse tell the patient? a. Her sexual partner should also be examined and treated. b. Her sexual partner must use a condom during intercourse. c. She should wear minipads to prevent reinfection as long as she has vaginal drainage. d. The vaginal suppository should be used in the morning so that it will be fighting the infection all day.

a. Gardnerella vaginalis infection is a bacterial vaginosis that may be sexually transmitted, and both partners may be infected. Treatment of the condition includes vaginal treatment with metronidazole (Flagyl) or clindamycin (Cleocin) or treatment via the oral route. Sexual activity is avoided until both partners are infection free. Minipads may be used to contain vaginal secretions, but they do not prevent reinfection. Vaginal suppositories and creams are used at bedtime so that the medication remains in the vagina for a long period of time.

24. When is a fat embolism most likely to occur? a. 24 to 48 hours after a fractured tibia b. 36 to 72 hours after a skull fracture c. 4 to 5 days after a fractured femur d. 5 to 6 days after a pelvic fracture

a. Initial manifestations of fat embolism usually occur 24 to 48 hours after injury. They are associated with fractures of long bones and multiple fractures related to pelvic injuries, including fractures of the femur, tibia, ribs, and pelvis. Venous thromboemboli (VTEs) tend to form later after injury of the extremities and pelvis.

12. What is a disadvantage of open reduction and internal fixation (ORIF) of a fracture compared to closed reduction? a. Infection b. Skin irritation c. Nerve impairment d. Complications of immobility

a. Open reduction involves use of a surgical incision to visualize the fracture and correct bone alignment. The main disadvantage is the risk of infection. Anesthesia complications are also possible. Preexisting medical conditions may have impact on surgical success. Skin irritation and nerve impairment are most likely with skin traction. Prolonged immobility is possible with skeletal traction.

8. The patient had a fracture. At 3 weeks to 6 months there is clinical union, and this is the first stage of healing that is sufficient to prevent movement of the fracture site when the bones are gently stressed. How is this stage of fracture healing documented? a. Ossification b. Remodeling c. Consolidation d. Callus formation

a. Ossification is the stage of fracture healing when there is clinical union and enough strength to prevent movement at the fracture site. Remodeling is the return to preinjury structural strength and shape. Consolidation is when the distance between bone fragments eventually closes and radiologic union first occurs. The callus formation stage appears by the end of the second week of injury, when minerals and new bone matrix are deposited in the osteoid produced in the granulation tissue stage.

What should the nurse include when teaching the patient with acute PID how to care for herself? a. Promote rest in semi-Fowler's position. b. Perform vaginal irrigations every 4 hours. c. Instruct the patient to use tampons to control vaginal drainage. d. The patient should ambulate frequently to promote drainage of exudate.

a. Physical rest in semi-Fowler's position promotes drainage of the pelvic cavity by gravity and may prevent the development of abscesses high in the abdomen. Coitus, douching, and tampon use should be avoided to prevent spreading infection upward from the vagina, although frequent perineal care should be performed to remove infectious drainage.

A woman who has been postmenopausal for 10 years calls the clinic because of vaginal bleeding. The nurse schedules a visit for the patient and informs her to expect to have which diagnostic procedure? a. An endometrial biopsy b. Abdominal radiography c. A laser treatment to the cervix d. Only a routine pelvic examination and Pap test

a. Postmenopausal vaginal bleeding is an early sign of endometrial cancer. When it occurs, a sample of endometrial tissue must be taken to exclude cancer. An endometrial biopsy can be done as an office procedure and is indicated in this case. Abdominal x-rays and Pap tests are not reliable tests for endometrial cancer. Laser treatment of the cervix is indicated only for cervical dysplasia.

*A nurse is planning discharge teaching on home safety for an older adult client who has osteoporosis. Which of the following information should the nurse include in the teaching?* Select all that apply a. Remove throw rugs in walkways b. Use prescribed assistive devices c. Remove clutter from the environment d. Wear soft-bottomed shoes e. Maintain lighting of doorway areas

a. Remove throw rugs in walkways b. Use prescribed assistive devices c. Remove clutter from the environment e. Maintain lighting of doorway areas

36. A patient who had a below-the-knee amputation is to be fitted with a temporary prosthesis. What is most important to teach the patient? a. Inspect the residual limb daily for irritation. b. Apply an elastic shrinker before applying the prosthesis. c. Perform range-of-motion (ROM) exercises to the affected leg 4 times a day. d. Apply alcohol to the residual limb every morning and evening to toughen the skin.

a. Skin breakdown on the residual limb can delay the use of a prosthesis, so the limb should be inspected every day for signs of irritation or pressure areas. A residual limb shrinker is an elastic stocking that is used to mold the limb in preparation for prosthesis use. A cotton residual limb sock is worn with the prosthesis. Range-of-motion (ROM) exercises are not necessary when the patient is using a prosthesis. No substances except water and mild soap should be used on the residual limb.

19. If surgery is needed, which procedure would the nurse first prepare the patient for in the presence of compartment syndrome? a. Fasciotomy b. Amputation c. Internal fixation d. Release of tendons

a. Soft tissue edema in the area of the injury may cause increased pressure within the closed tissue compartments formed by the nonelastic fascia, causing compartment syndrome. If symptoms occur, surgical incision of fascia may be needed (fasciotomy). Amputation is usually needed only if the limb becomes septic because of untreated compartment syndrome.

Priority Decision: To prepare a woman who has been raped for physical examination, what should the nurse do first? a. Ensure that a signed informed consent is obtained from the patient. b. Provide a private place for the patient to talk about what happened to her. c. Instruct the patient not to wash, eat, drink, or urinate before the examination. d. Administer prophylaxis for sexually transmitted infections (STIs) and tetanus.

a. Specific informed consent must be obtained from the rape victim before any examination can be made or rape data collected. Following consent, the patient is advised not to wash, eat, drink, or urinate before the examination so that evidence can be collected for medicolegal use. Prophylaxis for STIs, hepatitis B, and tetanus is administered following examination and follow-up testing for pregnancy and human immunodeficiency virus (HIV) is done in several weeks.

A 47-year-old patient who is experiencing andropause has decided to try the testosterone gel Testim. What should the nurse teach the patient and his wife about this gel? a. Wash the hands with soap and water after applying it. b. His wife should apply it to help him feel better about using it. c. Do not wear clothing over the area until it has been absorbed. d. The gel may be taken buccally if it is not effective on the abdomen.

a. The gel may spread the testosterone to others if it is not washed off of his hands after application. If his wife applies the gel, she should wear gloves to prevent absorption of the testosterone and its effects on her body. Clothing over the area until it has dried is recommended. The gel is only topical; a buccal testosterone tablet is called Striant.

A couple seeks assistance from an infertility specialist for evaluation of their infertility. What does the nurse inform the couple they can expect during the initial visit? a. Physical and psychosocial functioning examinations b. Assessment of tubal patency with a hysterosalpingogram c. Pelvic ultrasound for the woman and semen analysis for the man d. Postcoital testing to evaluate sperm numbers and motility in cervical and vaginal secretions

a. The initial visit of a couple seeking assistance with infertility includes a history and physical for both partners, psychosocial functioning, testing for medical problems and sexually transmitted infections (STIs), a cervical Papanicolaou (Pap) test, possible semen analysis, and instruction for at-home ovulation testing. A discussion of possible future testing options and cost is also done. If the couple decides to continue with treatment, further visits will include more intensive evaluation, including postcoital testing, a hysterosalpingogram, pelvic ultrasound, and midluteal progesterone and prolactin levels.

When teaching a patient with problems of pelvic support to perform Kegel exercises, what should the nurse tell the patient to do? a. Contract the muscles used to stop rectal gas expulsion. b. Tighten the lower abdominal muscles over the bladder area. c. Squeeze all of the perineal muscles as if trying to close the vagina. d. Lie on the floor and do leg lifts to strengthen the abdominal muscles.

a. The muscles that should be exercised are those affected by trying to stop an expulsion of gas from the rectum or stop urine midflow. Kegel exercises help to strengthen muscular support of the perineum, pelvic floor, and bladder and are also beneficial for problems with pelvic support and stress incontinence.

Which cancer is associated with intrauterine exposure to diethylstilbestrol (DES) or metastasis from another gynecologic cancer? a. Vaginal b. Ovarian c. Cervical d. Endometrial

a. Vaginal cancer is usually related to metastases of other cancers or intrauterine exposure to diethylstilbestrol (DES). Cervical, endometrial, and ovarian cancer have other or unknown causes.

*The increased risk for falls in the older adult is likely due to* Select all that apply a. changes in balance. b. decrease in bone mass. c. loss of ligament elasticity. d. erosion of articular cartilage. e. decrease in muscle mass and strength

a. changes in balance b. decreases in bone mass c. loss of ligament elasticity e. decrease in muscle mass and strength rationale: Aging can cause changes in a person's sense of balance, making the person unsteady, and proprioception may be altered. The risk for falls also increases in older adults partly because of a loss of muscle strength, loss of ligament elasticity, and decrease in bone density.

*A patient with a pelvic fracture should be monitored for* a. changes in urine output. b. petechiae on the abdomen. c. a palpable lump in the buttock. d. sudden increase in blood pressure

a. changes in urine output rationale: Pelvic fractures may cause serious intraabdominal injury, such as hemorrhage, and laceration of the urethra, bladder, or colon. Patients may survive the initial pelvic injury, only to die of sepsis, FES, or VTE. Because a pelvic fracture can damage other organs, assess bowel and urinary elimination and distal neurovascular status.

*A patient with a torn ligament in the knee asks what the ligament does. The nurse's response is based on the knowledge that ligaments* a. connect bone to bone. b. provide strength to muscle. c. lubricate joints with synovial fluid. d. relieve friction between moving parts

a. connect bone to bone rationale: Ligaments are composed of dense, fibrous connective tissue that contains bundles of closely packed collagen fibers arranged in the same plane for additional strength. They connect bone to bone providing stability for joints

*The nurse is obtaining a health history of a patient with a fracture. Which condition poses the most concern related to the musculoskeletal system?* a. Diabetes b. Hypertension c. Chronic bronchitis d. Nephrotic syndrome

a. diabetes rationale: Certain illnesses are known to affect the musculoskeletal system directly or indirectly. These diseases include tuberculosis, poliomyelitis, diabetes, parathyroid problems, hemophilia, rickets, soft tissue infection, and neuromuscular disabilities

*While performing passive range of motion for a patient, the nurse puts the elbow joint through the movements of* Select all that apply a. flexion and extension. b. inversion and eversion. c. pronation and supination. d. flexion, extension, abduction, and adduction. e. pronation, supination, rotation, and circumduction.

a. flexion and extension c. pronation and supination rationale: Movements that occur at the elbow include pronation, supination, flexion, and extension

*A patient with a humeral fracture is returning for a 4-week checkup. The nurse explains that initial evidence of healing on x-ray is indicated by* a. formation of callus. b. complete bony union. c. hematoma at the fracture site. d. presence of granulation tissue.

a. formation of callus rationale: Bone goes through a remarkable healing process (e.g., union) that occurs in stages. The third stage is callus formation. As minerals (e.g., calcium, phosphorus, and magnesium) and new bone matrix are deposited in the osteoid, an unorganized network of bone is formed that is woven about the fracture parts. Callus is composed primarily of cartilage, osteoblasts, calcium, and phosphorus. It usually appears by the end of the second week after injury. Evidence of callus formation can be verified on x-rays

*The nurse teaches the patient with an above-the-knee amputation that the residual limb should not be routinely elevated because this position promotes* a. hip flexion contracture. b. clot formation at the incision. c. skin irritation and breakdown. d. increased risk for wound dehiscence

a. hip flexion contracture rationale: Flexion contractures may delay the rehabilitation process after amputation. The most common and debilitating contracture is hip flexion. To prevent flexion contractures, the patient should avoid sitting in a chair for more than 1 hour with hips flexed or with pillows under the surgical extremity. Unless specifically contraindicated, help the patient lie on the abdomen for 30 minutes 3 or 4 times each day and position the hip in extension while prone.

*A patient who has had surgical correction of bilateral hallux valgus is being discharged from the same-day surgery unit. The nurse will teach the patient to* a. rest frequently with the feet elevated. b. wear shoes continually except when bathing. c. soak the feet in warm water several times a day. d. expect the feet to be numb for the next few days.

a. rest frequently with the feet elevated rationale: After surgical correction of bilateral hallux valgus, the feet should be elevated with the heel off the bed to reduce discomfort and decrease edema.

*Teach the patient with fibromyalgia the importance of limiting intake of which foods?* Select all that apply a. Sugar b. Alcohol c. Caffeine d. Red meat e. Root vegetables

a. sugar b. alcohol c. caffeine rationale: Dietitians often urge patients with fibromyalgia to limit their intake of sugar, caffeine, and alcohol because these substances have been shown to be muscle irritants

*A patient scheduled for a prostatectomy for prostate cancer expresses the fear that he will have erectile dysfunction. In responding to this patient, the nurse should keep in mind that* a. PD5 inhibitors are not recommended in prostatectomy patients b. erectile dysfunction can occur even with a nerve-sparing procedure c. the most common complication of this surgery is postoperative bowel incontinence d. the provider will place a penile implant during surgery to treat any dysfunction

b

*In assessing a patient for testicular cancer, the nurse understands that the manifestations of this disease often include* a. urinary frequency b. painless mass in the scrotal area c. erectile dysfunction with retrograde ejaculation d. rapid onset of dysuria with scrotal swelling and fever

b

*In telling a patient with infertility what she and her partner can expect, the nurse explains that* a. ovulatory studies can help determine tube patency b. a hysterosalpingogram is a common diagnostic study c. for most couples, the cause of infertility is usually not found d. semen analysis is performed only if testosterone levels are low

b

A patient with osteomyelitis has a nursing diagnosis of risk for injury. What is an appropriate nursing intervention for this patient a. Use careful and appropriate disposal of soiled dressings b. Gently handle the involved extremity during movement c. Measure the circumference of the affected extremity daily d. Range-of-motion exercise every 4 hours to the involved extremity.

b

Patient-Centered Care: A 24 year old patient with a 12 year history of Becker muscular dystrophy is hospitalized with heart failure. What is an appropraite nursing intervention for this patient. a. Feed and bathe the patient b. Reposition frequently to avoid skin and respiratory complications c. Provide hand weighs for the patient to exercise the upper extremities d. Use orthopedic braces to promote ambulations and prevent muscle wasting

b

Patient-Centered Care: A patient with chronic osteomyelitis has been hospitalized for a surgical debridement procedure. What does the nurse explain to the patient as the rationale for the surgical treatment. a. Removal of the infection prevents the need for bone and skin grafting b. Formation of scar tissue has led to protected area of bacterial growth c. The process of depositing new bone blacks the vascular supply to the bone d. Antibiotics are not effective against microorganisms that cause chronic osteomyelitis

b

Priority Decision: A laminectomy and spinal fusion are performed on a patient with a herniated lumbar intervertebral disc. During the postoperative period, which finding is of most concern to the nurse a. Paralytic ileus b. Urinary incontinence c. Greater pain at the graft site than at the lumber incision site d. Leg and arm movement and sensation unchanged from preoperative status

b

Priority Decision: During a follow-up visit to a patient with acute osteomyelitis treated with IV antibiotics, the home health nurse is told by the patient's wife that she can hardly get the a patient to eat because his mouth is so sore. In assessing the patient's mouth, what is the most likely finding that the nurse should expect to find a. A dry, cracked tongue with central furrow b. white, curdlike membranous lesions of the mucosa c. Ulcers of the mouth and lips surrounded by a reddened base d. Single or clustered vesicles on the tongue and buccal mucosa

b

Which statement describes osteosarcoma a. High rate of local recurrence. b. Very malignant and metastasizes early c. Arises in cancellous ends of long bones d. develops in the medullary cavity of long bones

b

Identify methods to specifically prevent osteoporosis in postmenopausal women (select all that apply) a. eating more beef b. eating 8 ounces of yogurt daily c. performing weight-bearing exercise d. spending 15 minutes in the sun each day e. taking postmenopausal estrogen replacement

b, c

Which therapies for BPH are done on an outpatient basis (select all that apply)? a. Intraprostatic urethral stents b. Transurethral needle ablation (TUNA) c. Transurethral incision of prostate (TUIP) d. Transurethral microwave therapy (TUMT) e. Visual laser ablation of the prostate (VLAP)

b, c, d. TUNA, TUIP, and TUMT are currently done on an outpatient basis or in a health care provider's office.

*A nurse is reviewing the medical record of a client who is menopausal. Which of the following findings should the nurse expect?* Select all that apply A. Increased vaginal secretion B. Decreased bone density C. Increased HDL level D. Decreased skin elasticity E. Increased pubic hair growth F. Decreased follicle stimulating hormone level

b, d

*The nurse should explain to the patient who has erectile dysfunction that* Select all that apply a. the most common cause is benign prostatic hyperplasia b. ED may be due to medications or conditions such as diabetes c. only men who are 65 years or older benefit from PDE5 inhibitors d. there are medications and devices that can be used to help with erections e. this condition is primarily due to anxiety and best treated with psychotherapy

b, d

Which female patients are at risk for developing osteoporosis (select all that apply)? a. 60-year-old white aerobics instructor b. 55-year-old Asian American cigarette smoker c. 62-year-old African American on estrogen therapy d. 68-year-old white who is underweight and inactive e. 58-year-old Native American who started menopause prematurely

b, d, e

*The nurse explains to the patient with chronic bacterial prostatitis who is undergoing antibiotic therapy that* Select all that apply a. all patients require hospitalization b. pain will lessen once treatment has ended c. course of treatment is generally 2-4 weeks d. long-term therapy may be indicated in immunocompromised patients e. if the condition is not treated appropriately, he is at risk for prostate cancer

b, d.

Which serological tests would be done to evaluate rheumatoid arthritis? A. Uric acid B. Anti-DNA antibody C. Rheumatoid factor(RF) D. Antinuclear antibody (ANA) E. Erythrocyte sedimentation rate (ESR) F. anticyclic citrullinated peptide (anti-CCP)

c, d, e, f

*A patient with osteomyelitis undergoes surgical debridement with implantation of antibiotic beads. When the patient asks why the beads are used, the nurse answers* Select all that apply a. "Oral or IV antibiotics are not effective in most cases of bone infection." b. "The beads are an adjunct to debridement and antibiotics for deep infections." c. "The beads are used to deliver antibiotics directly to the site of the infection." d. "This is the safest method to deliver long-term antibiotic therapy for bone infection." e. "Ischemia and bone death related to osteomyelitis are impenetrable to IV antibiotics."

b. "the beads are adjunct to debridement and antibiotics for deep infections" c. "the beads are used to deliver antibiotics directly to the site of infection" rationale: Treatment of chronic osteomyelitis includes surgical removal of the poorly perfused tissue and dead bone in addition to the extended use of IV and oral antibiotics. Antibiotic acrylic bead chains may be placed during surgery to help fight the infection

*A patient with osteosarcoma of the humerus shows understanding of his treatment options when he states* a. "I accept that I have to lose my arm with surgery." b. "The chemotherapy before surgery will shrink the tumor." c. "This tumor is related to the melanoma I had 3 years ago." d. "I'm glad they can take out the cancer with such a small scar."

b. "the chemotherapy before surgery will shrink the tumor." rationale: A patient with osteosarcoma usually has preoperative chemotherapy to decrease tumor size. This increases the chance that limb-salvage procedures, including wide surgical resection of the tumor, can be done. Osteosarcoma is an extremely aggressive primary bone tumor that rapidly metastasizes to distant sites.

17. Patient-Centered Care: A patient is discharged from the outpatient clinic after application of a synthetic fiberglass long arm cast for a fractured ulna. Before discharge, what instruction should the nurse provide to the patient? a. Never get the cast wet. b. Move the shoulder and fingers frequently. c. Place tape petals around the edges of the cast when it is dry. d. Use a sling to support the arm at waist level for the first 48 hours.

b. A patient with any type of cast should exercise the joints above and below the cast frequently. Moving the fingers will improve circulation and help prevent edema. Unlike plaster casts, thermoplastic resin or fiberglass casts are relatively waterproof. If they become wet, they can be dried with a hair dryer on low setting. Tape petals are used on plaster casts to protect the edges from breaking and crumbling but are not needed for synthetic casts. After the cast is applied, the extremity should be elevated at the level of the heart to promote venous return. Ice may be used to prevent edema.

A patient with a stage 0 cervical cancer identified from a Papanicolaou (Pap) test asks the nurse what this finding means. The nurse's response should include which information? a. Malignant cells have extended beyond the cervix to the upper vagina. b. Abnormal cells are present but are confined to the epithelial layer of the cervix. c. Atypical cells characteristic of inflammation but not necessarily malignancy are present. d. This is a common finding on Pap testing, and she will be examined frequently to see whether the abnormal cells spread beyond the cervix.

b. A stage 0 cervical cancer indicates cancer in situ that is confined to the epithelial layer of the cervix and requires treatment. Stage 0 is the least invasive. Stage I is confined to the cervix. Stage II has spread beyond the cervix to the upper two thirds of the vagina but not the tissues around the uterus. Stage III involves the pelvic wall, lower third of the vagina, and/or kidney problems. Stage IV indicates spread to distant organs.

The nurse provides discharge teaching to a patient following a TURP and determines that the patient understands the instructions when he makes which statement? a. "I should use daily enemas to avoid straining until healing is complete." b. "I should avoid heavy lifting, climbing, and driving until my follow-up visit." c. "At least I don't have to worry about developing cancer of the prostate now." d. "Every day I should drink 10 to 12 glasses of liquids such as coffee, tea, or soft drinks."

b. Activities that increase intraabdominal pressure should be avoided until the surgeon approves these activities at a follow-up visit. Stool softeners and high-fiber diets may be used to promote bowel elimination but enemas should not be used because they increase intraabdominal pressure and may initiate bleeding. Because TURP does not remove the entire prostate gland, the patient needs annual prostatic examinations to screen for cancer of the prostate. Fluid intake should be high but caffeine and alcohol should not be used because they have a diuretic effect and increase bladder distention.

2. The patient asks, "What does the doctor mean when he says that I have an avulsion fracture in my leg? I thought I had a sprain!" What is the best response by the nurse? a. "It is a fracture with more than 2 fragments." b. "It means that a ligament pulled a bone fragment loose." c. "The line of the fracture is twisted along the shaft of the bone." d. "The line of the fracture is at right angles to the longitudinal axis of the bone."

b. An avulsion fracture occurs when a ligament pulls a bone fragment loose. The pain is similar to a sprain. A fracture with 2 or more fragments is a comminuted fracture. A spiral fracture is twisted around a bone shaft. A transverse fracture occurs when the line of fracture is at right angles to the longitudinal axis.

What are characteristics of Paget's disease (select of that apply) a. Results from vitamin D deficiency b. loss of total bone mass and substance c. abnormal remodeling and resorption of bone d. Most common in bones of spine, hips, and wrists e. generalized bone decalcification with bone deformity f. replacement of normal marrow with vascular connective tissue

c, f

The patient is told by the healthcare provider that the size of the patients muscle has decreased. How should the nurse document this occurrence? a. hyaline b. atrophy c. isometric d. hypertrophy

b. Atrophy Rationale: Hyaline is the most common type of cartilage tissue. Isometric is a muscle contraction that produces hypertrophy. Hypertrophy is the increase in size of cells causing an increase in the organ

Before undergoing a TURP, what should the patient be taught? a. Some degree of urinary incontinence is likely to occur. b. This surgery results in some degree of retrograde ejaculation. c. Erectile dysfunction is a common complication of this prostate surgery. d. An indwelling catheter will be used to maintain urinary output until healing is complete.

b. Because of injury to the internal urinary sphincter, there is usually some degree of retrograde ejaculation following most transurethral surgeries, especially following TURP. The semen is ejaculated into the bladder and is eliminated with the next voiding. Urinary incontinence, erectile dysfunction, and continued catheterization are uncommon following TURP.

41. After a total knee arthroplasty, a patient has a continuous passive motion (CPM) machine for the affected joint. What should the nurse explain to the patient is the purpose of this device? a. To relieve edema and pain at the incision site b. To promote early joint mobility and increase knee flexion c. To prevent venous stasis and the formation of a deep venous thrombosis d. To improve arterial circulation to the affected extremity to promote healing

b. Continuous passive motion (CPM) machines may be used after knee surgery to promote early joint mobility. Because joint dislocation is not a problem with knee replacements, early ambulation to prevent deep vein thrombosis (DVT) and improve circulation, exercise with straight leg raises, and gentle ROM maybe encouraged postoperatively.

10. A patient is brought to the emergency department (ED) with an injured lower left leg following a fall while rock climbing. The nurse identifies the presence of a fracture based on what cardinal sign of fracture? a. Muscle spasms b. Obvious deformity c. Edema and swelling d. Pain and tenderness

b. Deformity is the cardinal sign of fracture but may not be apparent in all fractures. Other supporting signs include edema and swelling, localized pain and tenderness, muscle spasm, ecchymosis, loss of function, crepitation, and an inability to bear weight.

*The nurse should advise the woman recovering from surgical treatment of an ectopic pregnancy that* a. she has an increased risk for salpingitis b. bed rest must be maintained for 12 hours to assist in healing c. having one ectopic pregnancy increases her risk for another d. intrauterine devices and infertility treatments should be avoided

c.

40. When positioning the patient after a total hip arthroplasty with a posterior approach, it is important that the nurse maintain the affected extremity in what position? a. Adduction and flexion b. Abduction and extension c. Abduction and internal rotation d. Adduction and external rotation

b. During hospitalization, after a total hip arthroplasty with a posterior approach, an abduction pillow is placed between the legs to maintain abduction and the leg is extended. Extremes of internal rotation, adduction, and 90-degree flexion of the hip must be avoided for 4 to 6 weeks postoperatively to prevent dislocation of the prosthesis.

Priority Decision: A patient with abdominal pain and irregular vaginal bleeding is admitted to the hospital with a suspected ectopic pregnancy. Before actual diagnosis, what is the most appropriate action by the nurse? a. Provide analgesics for pain relief. b. Monitor vital signs, pain, and bleeding frequently. c. Offer support for the patient's emotional response to the loss of the pregnancy. d. Explain the need for frequent blood samples for β-human chorionic gonadotropin monitoring

b. Ectopic pregnancy is a life-threatening condition. If the fallopian tube ruptures, profuse bleeding can lead to hypovolemic shock. All of the interventions are indicated, but the priority is monitoring the vital signs and pain for evidence of bleeding.

A patient with a 10-week pregnancy is admitted to the emergency department (ED) with vaginal bleeding and abdominal cramping. What does the nurse recognize about this situation? a. The patient will recover quickly when the bleeding stops. b. The patient is most likely experiencing a spontaneous abortion. c. The patient will be scheduled for an immediate dilation and curettage (D&C). d. Treatment of the patient with bed rest is usually successful in preventing further bleeding.

b. In the presence of a confirmed pregnancy, uterine cramping with vaginal bleeding is the most important sign of spontaneous abortion. Other conditions causing vaginal bleeding, such as an incompetent cervix, do not usually cause cramping. There is no evidence that any medical treatment improves the outcome for spontaneous abortion. Blood loss can be significant, and the loss of the pregnancy may cause long-term grieving. Dilation and curettage (D&C) (if needed) is performed after the abortion to minimize blood loss and reduce the chance of infection.

On observation of the patient, the nurse notes the presence of a gait disturbance. How should the nurse further evaluate the patient? A. Palpate the hips for crepitation. B. Measure the length of the limbs. C. Evaluate the degree of leg movement. D. Compare the muscle mass of one leg with the other.

b. Measure the length of the limbs

42. Priority Decision: A patient with severe ulnar deviation of the hands undergoes an arthroplasty with reconstruction and replacement of finger joints. Postoperatively, what is it most important for the nurse to do? a. Position the fingers lower than the elbow at all times. b. Perform neurovascular assessments of the fingers every 2 to 4 hours. c. Encourage the patient to gently flex, extend, abduct, and adduct the fingers every 4 hours. d. Remind the patient that function of the hands is more important than cosmetic appearance.

b. Neurovascular checks following surgery are essential to detect compromised neurologic and vascular function caused by trauma or edema. Postoperatively, the hands are elevated with a bulky dressing in place. When the dressing is removed, a guided splinting program is started. After discharge, exercises are done 3 to 4 times each day. Before surgery, the patient is taught that the goal of the surgery is to restore function related to grasp, pinch, stability, and strength and the hands, not cosmetic appearance.

*To prevent or decrease age-related changes that occur after menopause in a patient who chooses not to take hormone therapy, the most important self-care measure to teach is* a. maintaining usual sexual activity b. increasing the intake of dairy products c. performing regular, aerobic, weight-bearing exercise d. taking vitamin E and B-complex vitamin supplements

c.

33. A patient reports pain in the foot of a leg that was recently amputated. What should the nurse recognize about this pain? a. It is caused by swelling at the incision. b. It should be treated with ordered analgesics. c. It will become worse with the use of a prosthesis. d. It can be managed with diversion because it is psychologic.

b. Phantom limb sensation or pain may occur after amputation, especially if pain was present in the affected limb preoperatively, and is a real sensation to the patient. It will first be treated with analgesics and other pain interventions (i.e., tricyclic antidepressants, antiseizure drugs, transcutaneous electrical nerve stimulation [TENS], mirror therapy, acupuncture). As recovery and ambulation progress, phantom limb sensation usually subsides.

*A nurse is providing care for a client who had a vertebroplasty of the thoracic spine. Which of the following actions should the nurse take?* a. Apply heat to the puncture site b. Place the client in a supine position c. Turn client every 1 hr d. Ambulate the client within the first hour post-procedure

b. Place the client in a supine position

6. Application of RICE (rest, ice, compression, elevation) is indicated for initial management of which type of injury? a. Muscle spasms b. Sprains and strains c. Repetitive strain injury d. Dislocations and subluxations

b. Rest, ice, compression, and elevation (RICE) are indicated to decrease edema resulting from sprains and some strains. Muscle spasms are usually treated with heat application and massage. Repetitive strain injuries require cessation of the precipitating activity and physical therapy. Dislocations or subluxations require immediate reduction and immobilization to prevent vascular impairment and bone cell death.

A young woman is admitted to the hospital with acute pelvic inflammatory disease (PID). During the nursing history, the nurse notes which risk factor as being most significant for this patient? a. Lack of any method of birth control b. Sexual activity with multiple partners c. Use of a vaginal sponge for contraception d. Recent antibiotic-induced monilial vaginitis

b. Sexual activity with multiple partners increases the risk for pelvic inflammatory disease (PID), and there is often a history of an acute infection of the lower genital tract caused by gonococcal or chlamydia microorganisms. The only significant contraceptive issue related to PID is that condom use will help to prevent STIs that may lead to PID.

Priority Decision: On admission of a victim of sexual assault to the ED, what should be the first priority of the nurse? a. Contact a rape support person for the patient. b. Assess the patient for urgent medical problems. c. Question the patient about the details of the assault. d. Inform the patient what procedures and treatments will be performed

b. Sexual assault is an act of violence and the first priority of care for the patient should be assessment and treatment of serious injuries involving extragenital areas, such as fractures, subdural hematomas, cerebral concussions, and intraabdominal injuries. All of the other options as well as preserving forensic evidence are appropriate treatments, but treatment for shock and urgent medical injuries is the first priority

Patient-Centered Care: A 51-yr-old woman suffered a wrist fracture when she slipped on the ice. She has her uterus and is interested in starting hormone replacement therapy (HRT), as she is also experiencing menopause symptoms. What should the nurse include when discussing the risks and benefits of HRT with this patient? a. Taking only progesterone is suggested for a woman with a uterus. b. Taking both estrogen and progesterone may decrease her bone loss. c. The risk of breast cancer and cardiovascular disease is decreased with HRT. d. Taking estrogen and progesterone will increase the risk of endometrial cancer.

b. Taking combination hormone replacement therapy (HRT) increases bone marrow density and decreases fractures. Both progesterone and estrogen are recommended for a menopausal woman with a uterus. Progesterone alone is not as effective as estrogen. The risk for breast cancer is increased, and the risk for endometrial cancer is decreased with combination HRT. Evidence does not support using HRT to prevent cardiovascular disease.

14. Priority Decision: A patient is admitted with an open fracture of the tibia after a bicycle accident. What question should the nurse ask when assessing the patient? a. Any previous injuries to the leg b. The status of tetanus immunization c. The use of antibiotics in the last month d. Whether the injury was exposed to dirt or gravel

b. Tetanus prevention is always indicated if the patient has not been immunized or does not have a current booster. Infection is the greatest risk with an open fracture, and all open fractures are considered contaminated. Although prophylactic antibiotics are used in management of open fractures, neither recent antibiotic therapy nor previous injury to the site is relevant. Dirt or gravel contamination will be evident on physical assessment.

Priority Decision: A patient with continuous bladder irrigation following a prostatectomy tells the nurse that he has bladder spasms and leaking of urine around the catheter. What should the nurse do first? a. Slow the rate of the irrigation. b. Assess the patency of the catheter. c. Encourage the patient to try to urinate around the catheter. d. Administer a belladonna and opium (B&O) suppository as prescribed.

b. The nurse should first check for the presence of clots obstructing the catheter or tubing and then may administer a belladonna and opium (B&O) suppository if one is ordered. The patient should not try to void around the catheter because this will increase the spasms. The flow rate of the irrigation fluid may be decreased if orders permit because fast-flowing, cold fluid may also contribute to spasms.

A 20-yr-old patient with PID is crying and tells the nurse that she is afraid she will not be able to have children as a result of the infection. What is the nurse's best response to this patient? a. "I would not worry about that now. Our immediate concern is to cure the infection you have." b. "PID increases the possibility of infertility. Would you like to talk about what it means to you?" c. "Sterility following PID is possible but not common, and it is too soon to know what the effects will be." d. "The infection can cause more serious complications, such as abscesses and shock that you should be more concerned about."

b. The risk for infertility following PID is high, and the nurse should allow time for the patient to express her feelings, clarify her concerns, and begin problem solving with regard to the outcomes of the disease. Responses that do not allow for discussion of feelings and concerns and that tell the patient how she should feel or what she should worry about are not therapeutic.

23. In a patient with a stable vertebral fracture, what should the nurse teach the patient? a. Remain on bed rest until the pain is gone. b. Logroll to keep the spine straight when turning. c. How to use bone cement to correct the problem. d. Take as much analgesic as needed to relieve the pain.

b. The spine should be kept straight by turning the shoulders and hips together (logrolling). This keeps the spine in good alignment until union has been accomplished. Bed rest may be required for a short time but not until the pain is gone. Bone cement is used by the surgeon to stabilize vertebral compression fractures. Analgesics should be taken only as ordered. If they do not manage the pain, the HCP should be notified.

Which treatment for BPH uses a low-wave radiofrequency to precisely destroy prostate tissue? a. Laser prostatectomy b. Transurethral needle ablation (TUNA) c. Transurethral microwave thermotherapy (TUMT) d. Transurethral electrovaporization of prostate (TUVP)

b. The transurethral needle ablation (TUNA) uses lowwave radiofrequency to heat the prostate, causing necrosis. Laser prostatectomy uses a laser beam. Transurethral microwave thermotherapy (TUMT) uses microwave radiating heat to produce coagulative necrosis of the prostate and is not used for men with rectal problems. Transurethral electrovaporization of prostate (TUVP) uses electrosurgical vaporization and desiccation to destroy prostate tissue.

*When administering medications to the patient with chronic gout, the nurse recognizes which drug is used as a treatment for this disease?* a. Colchicine b. Allopurinol c. Sulfasalazine d. Cyclosporine

b. allopurinol rationale: Febuxostat (Uloric), a selective inhibitor of xanthine oxidase, is given for long-term management of hyperuricemia in persons with chronic gout. An acute episode of gout is treated with colchicine and NSAIDs.

*In assessing the joints of a patient with osteoarthritis, the nurse understands that Bouchard's nodes* a. are often red, swollen, and tender. b. indicate osteophyte formation at the PIP joints. c. are the result of pannus formation at the DIP joints. d. occur from deterioration of cartilage by proteolytic enzymes.

b. indicate osteophyte formation at the PIP joints rationale: Bouchard's nodes are bony deformities of the proximal interphalangeal joints that indicate osteophyte formation and loss of joint space in osteoarthritis.

*A nurse is providing dietary teaching about calcium-rich foods to a client to has osteoporosis. Which of the following foods should the nurse include in the instructions?* a. white bread b. kale c. apples d. brown rice

b. kale

*A nurse is admitting an older adult client who has suspected osteoporosis. Which of the following findings are risk factors for osteoporosis?* Select all that apply a. history of consuming 1 glass of wine daily b. loss in height of 2 in. (5.1 cm) c. body mass index (BMI) of 18 d. kyphotic curve at upper thoracic spine e. history of lactose intolerance

b. loss in height of 2 in. (5 cm) c. body mass index (BMI) of 21 d. kyphotic curve at upper thoracic spine e. history of lactose intolerance

*A patient with rheumatoid arthritis has articular involvement. The nurse recognizes these characteristic changes include* Select all that apply a. bamboo-shaped fingers. b. metatarsal head dislocation in feet. c. noninflammatory pain in large joints. d. asymmetric involvement of small joints. e. morning stiffness lasting 60 minutes or more.

b. metatarsal head dislocation in feet e. morning stiffness lasting 60 minutes or more rationale: Morning stiffness may last from 60 minutes to several hours or more, depending on disease activity. Metatarsal head dislocation and subluxation in the feet may cause pain and walking disability. Joint symptoms occur symmetrically and often affect the small joints of the hands (proximal interphalangeal [PIP] and metacarpophalangeal [MCP] joints) and feet (metatarsophalangeal [MTP] joints). Larger peripheral joints such as the wrists, elbows, shoulders, knees, hips, ankles, and jaw may be involved. Rheumatoid arthritis is an inflammatory disorder. In early disease, the fingers may become spindle-shaped from synovial hypertrophy and thickening of the joint capsule.

*The nurse should teach the patient with ankylosing spondylitis the importance of* a. avoiding extremes in environmental temperatures b. regularly exercising and maintaining proper posture. c. maintaining patient's usual physical activity during flares. d. applying hot and cool compresses for relief of local symptoms.

b. regularly exercising and maintaining proper posture rationale: Patients with AS should exercise after pain and stiffness are managed. Teach the patient with AS about regular exercise and attention to posture, local moist-heat applications, and knowledgeable use of drugs. Postural control is important for minimizing spinal deformity. Proper positioning at rest is essential. Postural training emphasizes avoiding spinal flexion (e.g., leaning over a desk), heavy lifting, and prolonged walking, standing, or sitting. Exercise should include back, neck, and chest stretches. Discourage excessive physical exertion during periods of active flare-up of the disease. The mattress should be firm. The patient should sleep on the back with a flat pillow, avoiding positions that encourage flexion deformity.

*A patient with osteoarthritis is scheduled for total hip arthroplasty. The nurse explains the purpose of this procedure is to* Select all that apply a. fuse the joint. b. replace the joint. c. prevent further damage. d. improve or maintain ROM. e. decrease the amount of destruction in the joint.

b. replace the joint d. improve or maintain ROM rationale: Arthroplasty is the reconstruction or replacement of a joint. This surgical procedure is done to relieve pain, improve or maintain range of motion, and correct deformity. Total hip arthroplasty (THA) provides significant pain relief and improved function for a patient with osteoarthritis (OA).

*What is most important to include in the teaching plan for a patient with osteopenia?* a. Lose weight. b. Stop smoking. c. Eat a high-protein diet. d. Start swimming for exercise

b. stop smoking rationale: Patients with osteopenia should be taught to quit smoking to decrease bone loss.

*In caring for a patient after a spinal fusion, the nurse would report which finding to the health care provider?* a. The patient has a single episode of emesis. b. The patient is unable to move the lower extremities. c. The patient is nauseated and has not voided in 4 hours. d. The patient reports of pain at the bone graft donor site.

b. the patient is unable to move the lower extremities rationale: After spinal fusion surgery, the nurse should frequently monitor peripheral neurovascular condition. Movement of the arms and legs and assessment of sensation should be no worse in comparison with the preoperative status. These assessments are repeated at least every 2 to 4 hours during the first 48 hours after surgery, and findings are compared with the preoperative assessment. The nurse should immediately report any new muscle weakness to the HCP and record this in the patient's medical record.

*An abnormal assessment finding of the musculoskeletal system is* a. equal leg length bilaterally. b. ulnar deviation and subluxation. c. full range of motion in all joints. d. muscle strength of 5/5 in all muscles.

b. ulnar deviation and subluxation rationale: Normal physical assessment findings of the musculoskeletal system include normal and full range of motion; equal leg length bilaterally, and muscle strength score of 5/5. Ulnar deviation and subluxation are associated with rheumatoid arthritis

Serum tumor markers that may be elevated on diagnosis of testicular cancer and used to monitor the response to therapy include a. tumor necrosis factor (TNF) and C-reactive protein (CRP). b. α-fetoprotein (AFP) and human chorionic gonadotropin (hCG). c. prostate-specific antigen (PSA) and prostate acid phosphatase (PAP). d. carcinoembryonic antigen (CEA) and antinuclear antibody (ANA).

b. α-Fetoprotein (AFP) and human chorionic gonadotropin (hCG) are glycoproteins that may be elevated in testicular cancer. If they are elevated before surgical treatment, the levels are noted, and if response to therapy is positive, the levels will decrease. Lactate dehydrogenase (LDH) may also be elevated. Tumor necrosis factor (TNF) is a normal cytokine responsible for tumor surveillance and destruction. C-reactive protein (CRP) is found in inflammatory conditions and widespread malignancies. PSA and PAP are used for screening of prostatic cancer. Carcinoembryonic antigen (CEA) is a tumor marker for cancers of the GI system. Antinuclear antibody (ANA) is found most frequently in autoimmune disorders.

*A nurse in a provider's office is reviewing a client's laboratory results, which shows a positive rapid plasma regain (RPR). Which of the following tests confirm the diagnosis of syphilis?* A. Venereal disease research laboratory (VDRL) B. D-dimer C. Fluorescent treponemal antibody absorbed (FTA-ABS) D. Sickledex

c

*Postoperatively, a patient has had a laser prostatectomy has continuous bladder irrigation with a three-way urinary catheter with a 30-mL balloon. When he complains of bladder spasms with the catheter in place, the nurse should* a. deflate the catheter balloon to 10mL to decrease bulk in the bladder b. deflate the catheter balloon and then reinflate to ensure it is patent c. explain that this feeling is normal and he should not try to urinate around the catheter d. stop the irrigation, assess the patient's vital signs, and notify the HCP of possible obstruction

c

A patient who experienced an open fracture of humerus 2 weeks ago is having increased pain at the fracture site. To identify a possible causative agent of osteomyelitis at the site, what should the nurse expect testing to include a. x-rays b. CT scan c. Bone Biopsy d. WBC count and erythrocyte sedimentation rate (ESR)

c

In promoting healthy feet, the nurse should recognize which factor is associated with most foot problems a. poor foot hygiene b. congential deformities c. improperly fitting shoes d. peripheral vascular disease

c

Priority Decision: Before re-positioning the patient on the side after a lumbar laminectomy, what should be the nurse's first action a. raise the head of the bed 30 degrees b. Have the patient flex the knees and hips c. Place a pillow between the patient's legs d. Have the patient grasp the side rail on the opposite side of the bed

c

*Postoperative nursing care for the woman with a gynecologic fistula includes* Select all that apply a. bed rest b. bladder training c. warm sitz baths d. perineal hygiene e. use of daily enemas

c, d

*In caring for a patient with endometriosis, the nurse teaches the patient that interventions used to treat or cure this condition may include* Select all that apply a. radiation b. antibiotic therapy c. oral contraceptive pills d. surgical removal of tissue e. total abdominal hysterectomy and salpingo-oophorectomy

c, d, e

21. A woman with osteoporosis slipped on the ice and now her wrist hurts. If there is a fracture, what type of fracture is expected? a. Dislocation b. Open fracture c. Colles' fracture d. Incomplete fracture

c. A Colles' fracture most often occurs in patients over 50 years of age with osteoporosis, frequently when the patient attempts to break a fall with an outstretched arm and hand. Dislocation is the complete separation of articular surfaces of the joint caused by a ligament injury. Open fracture occurs when there is communication with the external environment. A fracture is incomplete if only part of the bone shaft is fractured and the bone is still in 1 piece.

20. Which type of fracture can cause radial nerve and brachial artery damage and is reduced with a hanging arm cast? a. Fractured tibia b. Colles' fracture c. Fractured humerus d. Femoral shaft fracture

c. A fractured humerus may cause radial nerve and brachial artery damage. It may be reduced nonsurgically with a hanging arm cast. A fractured tibia and femur are in the leg. The Colles' fracture is in the wrist and manifests with pronounced swelling and obvious deformity of the wrist. It is treated with closed manipulation and immobilization. 21.

3. The patient with osteoporosis had a spontaneous hip fracture. How should the nurse document this before the x-ray results return? a. Open fracture b. Oblique fracture c. Pathologic fracture d. Greenstick fracture

c. A pathologic fracture is a spontaneous fracture at the site of bone disease, such as osteoporosis. An open fracture occurs when there is communication with the external environment. The oblique fracture has a slanted fracture line. A greenstick fracture is splintered on the convex side, and the other side is in intact with a concave bend.

A disturbed body image needing nursing interventions to assist the patient and family to cope may be seen in any patient undergoing gynecologic surgery. With which surgery will this most likely be expected to occur? a. Vaginectomy b. Hemivulvectomy c. Pelvic exenteration d. Radical hysterectomy

c. A pelvic exenteration is the most radical gynecologic surgery and results in removal of the uterus, ovaries, fallopian tubes, vagina, bladder, urethra, and pelvic lymph nodes and, in some situations, also the descending colon, rectum, and anal canal. There are urinary and fecal diversions on the abdominal wall, the absence of a vagina, and the onset of menopausal symptoms, all of which result in severe altered body structure and changes in body image. The patient and family will need much understanding and support during the long recovery period, including verbalization of feelings.

Priority Decision: When caring for a patient following a radical prostatectomy with a perineal approach, what is the priority nursing intervention the nurse should use to prevent complications? a. Use chemotherapeutic agents to prevent metastasis. b. Administer sildenafil (Viagra) as needed for erectile dysfunction. c. Provide wound care after each bowel movement to prevent infection. d. Insert a smaller indwelling urinary catheter to prevent urinary retention.

c. A prostatectomy performed with a perineal approach has a high risk for infection because of the proximity of the wound to the anus, so wound care is the priority. Chemotherapy is usually not the first choice of drug therapy following surgery, nor is sildenafil. The catheter size would not be changed but the catheter would be removed. Urinary incontinence is a bigger problem than retention.

A 44-yr-old woman undergoing a total abdominal hysterectomy asks whether she will need to take estrogen until she reaches the age of menopause. What is the best response by the nurse? a. "Yes, it will help to prevent the more intense symptoms caused by surgically induced menopause." b. "You are close enough to normal menopause that you probably won't need additional estrogen." c. "Because your ovaries won't be removed, they will continue to secrete estrogen until your normal menopause." d. "There are so many risks associated with estrogen replacement therapy that it is best to begin menopause now."

c. A total hysterectomy involves the removal of the uterus and cervix, but the fallopian tubes and ovaries are left intact. Although menstruation is terminated, normal ovarian production of estrogen continues. When the uterus, tubes, and ovaries are removed, it is called a total hysterectomy and bilateral salpingo-oophorectomy (TAH-BSO).

The patient is describing a feeling of something coming down her vagina and having a backache. What is most likely the cause of this discomfort? a. Cystocele b. Dysmenorrhea c. Uterine prolapse d. Abdominal distention

c. A uterine prolapse occurs when the uterus is displaced through the vagina, causing the feeling of something coming down her vagina, a backache, dyspareunia, or a heavy feeling in the pelvis

28. A patient with an extracapsular hip fracture is admitted to the orthopedic unit and placed in Buck's traction. How should the nurse explain the purpose of the traction to the patient? a. Pulls bone fragments back into alignment b. Immobilizes the leg until healing is complete c. Reduces pain and muscle spasms before surgery d. Prevents damage to the blood vessels at the fracture site

c. Although surgical repair is the preferred method of managing intracapsular and extracapsular hip fractures, patients may be treated initially with skin traction (e.g., Buck's traction) to relieve the painful muscle spasms before surgery is performed. Prolonged traction would be required to reduce the fracture or immobilize it for healing, creating a very high risk for complications of immobility.

38. When the nursing student asks the registered nurse (RN) what an arthroplasty is, what is the best description the RN can give the student? a. Surgical fusion of a joint to relieve pain b. Correction of bone deformity by removal of a wedge or slice of bone c. Reconstruction or replacement of a joint to relieve pain and correct deformity d. Used in rheumatoid arthritis to remove the tissue involved in joint destruction

c. An arthroplasty is reconstruction or replacement of a joint to relieve pain and correct deformity, especially with osteoarthritis, RA, avascular necrosis, congenital deformity, or dislocation. Arthrodesis is the surgical fusion of a joint to relieve pain. An osteotomy removes a wedge of bone to correct a bone deformity. Synovectomy is used in RA to remove the tissue involved in joint destruction.

The patient arrives at the urgent care facility worried about the fishy smell of her vaginal discharge. What does the nurse suspect will be diagnosed? a. Cervicitis b. Trichomoniasis c. Bacterial vaginosis d. Vulvovaginal candidiasis

c. Bacterial vaginosis is characterized by watery vaginal discharge with a fishy odor. Cervicitis displays mucopurulent discharge and postcoital spotting. Trichomoniasis has frothy greenish or gray discharge. Vulvovaginal candidiasis has thick, white, curdy discharge.

On admission to the ambulatory surgical center, a patient with BPH informs the nurse that he is going to have a laser treatment of his enlarged prostate. The nurse plans patient teaching with the knowledge that the patient will need to know what? a. The effects of general anesthesia b. The possibility of short-term incontinence c. Home management of an indwelling catheter d. Monitoring for postoperative urinary retention

c. Because of edema, urinary retention, and delayed sloughing of tissue that occurs with a laser prostatectomy, the patient will have postprocedure catheterization for up to 7 days. The procedure is done under local anesthetic, and incontinence or urinary retention is not usually a problem with laser prostatectomy.

Following a TURP, a patient has continuous bladder irrigation. Four hours after surgery, the catheter is draining thick, bright red clots and tissue. What should the nurse do? a. Release the traction on the catheter. b. Manually irrigate the catheter until the drainage is clear. c. Increase the rate of the irrigation and take the patient's vital signs. d. Clamp the drainage tube and notify the patient's health care provider.

c. Bleeding and blood clots from the bladder are expected after prostatectomy and continuous irrigation is used to keep clots from obstructing the urinary tract. The rate of the irrigation may be titrated to keep the clots from forming, if ordered, but the nurse should also check the vital signs because hemorrhage is the most common complication of prostatectomy. The traction on the catheter applies pressure to the operative site to control bleeding and should be relieved only if specific orders are given. The catheter does not need to be manually irrigated unless there are signs that the catheter is obstructed and clamping the drainage tube is contraindicated because it would cause distention of the bladder.

5. The athlete comes to the clinic with bursitis. What does the nurse know happens to the tissue to cause pain when bursitis occurs? a. Tearing of a ligament b. Stretching of muscle and fascia sheath c. Inflammation of synovial membrane sac at friction sites d. Incomplete separation of articular surfaces of joint caused by ligament injury

c. Bursitis is inflammation of the synovial membrane sac at friction sites. Tearing of a ligament is a sprain. Stretching of muscle and fascia sheath is a strain. Incomplete separation of articular surfaces of joints caused by ligament injury is subluxation.

When taking a nursing history from a patient with BPH, the nurse would expect the patient to report a. nocturia, dysuria, and bladder spasms. b. urinary frequency, hematuria, and perineal pain. c. urinary hesitancy, postvoid dribbling, and weak urinary stream. d. urinary urgency with a forceful urinary stream and cloudy urine.

c. Classic symptoms of uncomplicated BPH are those associated with irritative symptoms, including nocturia, frequency, urgency, dysuria, bladder pain, and incontinence associated with inflammation or infection. Urinary obstruction symptoms include diminished caliber and force of the urinary stream, hesitancy, difficulty initiating voiding, intermittent urination, dribbling at the end of urination, and a feeling of incomplete bladder emptying because of urinary retention.

Fertility and normal reproductive function can be maintained when a cancer of the cervix is successfully treated with which therapy? a. External radiation therapy b. Internal radiation implants c. Conization or laser surgery d. Cryotherapy or subtotal hysterectom

c. Conization (an excision of a cone-shaped section of the cervix) and laser treatment both are effective to locally remove or destroy malignant cells of the cervix and preserve fertility. Radiation treatments frequently impair ovarian and uterine function and lead to sterility. A subtotal hysterectomy would be contraindicated in the treatment of cervical cancer because the cervix would be left intact in this procedure.

37. Which joint surgery is used to arthroscopically remove devitalized tissue in joints? a. Osteotomy b. Arthrodesis c. Debridement d. Synovectomy

c. Debridement removes devitalized tissue from joints. Osteotomy corrects bone deformity by removal of a wedge or slice of bone. Arthrodesis surgically fuses a joint to relieve pain. Synovectomy removes tissue involved in joint destruction from rheumatoid arthritis (RA).

An infertile couple has used at-home ovulation testing using basal body temperature without conceiving. The nurse understands that what will be used first to treat this infertile couple? a. Surgery to reduce endometriosis b. Intrauterine insemination with sperm from the husband c. Selective estrogen receptor modulator (clomiphene [Clomid]) d. Assisted reproductive technologies (e.g., in vitro fertilization [IVF])

c. Drug therapy will be used before more invasive treatments. Drugs may include selective estrogen receptor modulators, menotropin (human menopausal gonadotropin), follicle-stimulating hormone agonists, gonadotropin-releasing hormone (GnRH) antagonists, GnRH agonists, or human chorionic gonadotropin (hCG). If the husband's reproductive system is functioning, intrauterine insemination with his sperm may later be done. The assisted reproductive technologies may be used if this is not successful. The surgery for endometriosis could be done if this was diagnosed, but that is not included in this question.

A patient with severe joint immobility is receiving physical and exercise therapy. To evaluate the effect of the treatment the nurse may assess joint range of motion with what equipment? A. Ergometer B. Myometer C. Goniometer D. Peak flow meter

c. Goniometer

What describes hypospadias? a. Scrotal lymphedema b. Undescended testicle c. Ventral urinary meatus d. Inflammation of the prepuce

c. Hypospadias is the urethral meatus located on the ventral surface of the penis. Scrotal lymphedema is called a hydrocele. An undescended testicle is cryptorchidism. Inflammation of the prepuce or foreskin is called phimosis.

A 55-year-old man with a history of prostate cancer in his family asks the nurse what he can do to decrease the risk of prostate cancer. What should the nurse teach him about prostate cancer risks? a. Nothing can decrease the risk because prostate cancer is primarily a disease of aging. b. Treatment of any enlargement of the prostate gland will help to prevent prostate cancer. c. Substituting fresh fruits and vegetables for high-fat foods in the diet may lower the risk of prostate cancer. d. Using a natural herb, saw palmetto, has been found to be an effective protection against prostate cancer.

c. Most prostate cancers (about 75%) are considered sporadic. About the only modifiable risk factor for prostate cancer is its association with a diet high in red and processed meat and high-fat dairy products along with a low intake of vegetables and fruits. Age, ethnicity, and family history are risk factors for prostate cancer but are not modifiable. Simple enlargement or hyperplasia of the prostate is not a risk factor for prostate cancer.

16. How should the nurse assess the neurologic status of the patient with a fractured humerus? a. Have the patient evert, invert, dorsiflex, and plantar flex the foot. b. Assess the location, quality, and intensity of pain below the site of the injury. c. Have the patient abduct the fingers, oppose the thumb and fingers, and flex and extend the wrist. d. Assess the color, temperature, capillary refill, peripheral pulses, and edema in the extremity.

c. Neurologic assessment includes evaluation of sensation, motor function, and pain in the upper extremity. Ask the patient to abduct the fingers (ulnar nerve), oppose the thumb and small fingers (median nerve), and flex and extend the wrist (or fingers if in a cast) (radial nerve). The nurse will assess pain and sensory function in the fingers. Evaluation of the feet would occur in lower extremity injuries. Assessment of color, temperature, capillary refill, peripheral pulses, and edema evaluates vascular condition.

18. Patient-Centered Care: A patient with a fractured tibia accompanied by extensive soft tissue damage initially has a splint applied and held in place with an elastic bandage. What early sign would alert the nurse that the patient is developing compartment syndrome? a. Paralysis of the toes b. Absence of peripheral pulses c. Distal pain unrelieved by opioid analgesics d. Skin over the injury site is blanched when the bandage is removed

c. Pain that is distal to the injury and unrelieved by opioid analgesics is the earliest sign of compartment syndrome; paresthesia is also an early sign. Paralysis and absence of peripheral pulses will eventually occur if there is no treatment, but these are late signs that often appear after permanent damage has occurred. The overlying skin may appear normal because the surface vessels are not occluded.

25. What assessment findings distinguish a fat embolism from a pulmonary embolism in a patient with a fracture? a. Tachycardia and dyspnea b. A sudden onset of chest pain c. Petechiae around the neck and upper chest d. Electrocardiographic (ECG) changes and decreased partial pressure of oxygen in arterial blood (PaO2)

c. Patients with fractures are at risk for both fat embolism and pulmonary embolism from VTE, but there is a difference in the time of occurrence. Fat embolism occurs shortly after the injury and thrombotic embolism occurring several days after immobilization. They both may cause pulmonary symptoms of chest pain, tachypnea, dyspnea, apprehension, tachycardia, and cyanosis. However, only fat embolism may cause petechiae found around the neck, anterior chest wall, axilla, buccal membrane of the mouth, and conjunctiva of the eye.

The patient has a burning sharp pain on the soul of the foot especially in the morning. The nurse knows this describes what common musculoskeletal problem? A. Pes planus B. Tenosynovitis C. Plantar fasciitis D. Muscle atrophy

c. Plantar fasciitis

In report, the nurses told that the patient has a contracture of the right arm. What does the nurse know this means? A. A fluid- filled cyst B. Generalized muscle pain C. Shortening of a muscle or ligament D. Grating sensation between bones with movement

c. Shortening of a muscle or ligament

32. Priority Decision: Twenty-four hours after a below-the-knee amputation, a patient uses the call system to tell the nurse that his dressing (a compression bandage) has fallen off. What should be the nurse's first action? a. Apply ice to the site. b. Cover the incision with dry gauze. c. Reapply the compression dressing. d. Elevate the extremity on 2 pillows.

c. The compression dressing or bandage supports the soft tissues, reduces edema, hastens healing, minimizes pain, and promotes residual limb shrinkage. If the dressing is left off, edema will form quickly and may delay rehabilitation. Elevation and ice will not be as effective at preventing the edema that will form. Dressing the incision with dry gauze will not provide the benefits of a compression dressing.

31. When preparing a patient for discharge after intermaxillary fixation of a mandibular fracture, which statement indicates that patient teaching has been successful? a. "I can keep my mouth moist by sucking on hard candy." b. "I should cut the wires with scissors if I begin to vomit." c. "I may use a bulk-forming laxative if my liquid diet causes constipation." d. "I should use a moist swab to clean my mouth every time I eat something."

c. The low-bulk, high-carbohydrate liquid diet and intake of air through a straw after mandibular fixation often lead to constipation and flatus, which may be relieved with bulk-forming laxatives, prune juice, and ambulation. Hard candy should not be held in the mouth. Wires or rubber bands should be cut only in the case of cardiac or respiratory arrest, and the patient should be taught to clear the mouth of vomitus or secretions. The mouth should be thoroughly cleaned with water, saline, or alkaline mouthwashes or using a Water Pik as necessary to remove food debris.

What is the rationale for the regular use of nonsteroidal antiinflammatory drugs (NSAIDs) during the first several days of the menstrual period for women who have primary dysmenorrhea? a. They suppress ovulation and the production of prostaglandins that occur with ovulation. b. They cause uterine relaxation and small vessel constriction, preventing cramping and abdominal congestion. c. They inhibit the production of prostaglandins believed to be responsible for menstrual pain and associated symptoms. d. They block the release of luteinizing hormone, preventing the increase in progesterone associated with maturation of the corpus luteum.

c. The release of excess prostaglandins from the endometrium at the time of menstruation or increased sensitivity to the prostaglandins is responsible for symptoms of primary dysmenorrhea and drugs that inhibit prostaglandin production and release, such as nonsteroidal antiinflammatory drugs (NSAIDs), are effective in many patients with primary dysmenorrhea. Oral contraceptives may be used for primary dysmenorrhea by reducing endometrial hyperplasia.

The patient is suspected of having endometriosis and/or uterine leiomyoma. What best describes what is found with these conditions? a. Endometriosis and uterine leiomyoma are two gynecologic conditions that increase in incidence with the onset of menopause. b. Danazol and Lupron (GnRH analog) are used to treat endometriosis and leiomyomas to create a pseudopregnancy. c. Treatment of endometriosis and leiomyomas depends on the severity of symptoms and the woman's desire to maintain her fertility. d. The presence of ectopic uterine tissue that bleeds and causes pelvic and abdominal adhesions, cysts, and pain is known as uterine leiomyoma.

c. The treatment of endometriosis and leiomyomas is surgical when the patient does not tolerate the symptoms, with the type of surgery for endometriosis dependent on the desire for pregnancy. Endometriosis and leiomyomas subside with the onset of menopause. Therefore the medications to treat them create a pseudomenopause. The ectopic uterine tissue is endometriosis, while leiomyomas are fibrous smooth muscle tumors.

The nurse teaches the patient having a vasectomy that what occurs after the procedure? a. The amount of ejaculate will be noticeably decreased. b. He may have difficulty maintaining an erection for several months. c. An alternative form of contraception must be used for 6 to 8 weeks. d. The testes will gradually decrease production of sperm and testosterone.

c. Until sperm distal to the anastomotic site is ejaculated or absorbed by the body, the semen will contain sperm and alternative contraceptive methods must be used. When a postoperative semen examination reveals no sperm, the patient is considered sterile. Following vasectomy, there is rarely noticeable difference in the amount of ejaculate because ejaculate is primarily seminal fluid. Vasectomy does not cause erectile dysfunction, nor does it affect testicular production of sperm or hormones.

The extent of urinary obstruction caused by BPH can be determined by which diagnostic study? a. A cystometrogram b. Transrectal ultrasound c. Urodynamic flow studies d. Postvoiding catheterization

c. Urinary flow meters are used to measure the urinary flow rate, which is slowed with increased obstruction. Cystourethroscopy may also evaluate the degree of obstruction but a cystometrogram measures bladder tone. A transrectal ultrasound may determine the size and configuration of the prostate gland. Post voiding catheterization measures residual urine.

When teaching a patient with premenstrual dysphoric disorder (PMDD) about management of the disorder, the nurse includes the need to a. limit dietary intake of caffeine and refined sugar. b. use estrogen supplements during the luteal phase. c. supplement the diet with vitamin B6, calcium, and magnesium. d. limit exercise and physical activity when symptoms are present.

c. Vitamin B6, calcium, and magnesium are recommended, as well as foods high in tryptophan, which may promote serotonin production, to improve mood changes. Limitation of refined sugar and caffeine in the diet may decrease the PMS symptoms of abdominal bloating, increased appetite, and irritability. Estrogen is not used during the luteal phase, but progesterone may be tried. Exercise is encouraged because it increases the release of endorphins, which elevate the mood, and has a tranquilizing effect on muscle tension.

A young woman who runs vigorously as a form of exercise has not had a menstrual period in more than 6 months. What should the nurse teach her? a. Normal periods will return when she stops running. b. Uterine balloon therapy may be necessary to promote uterine sloughing of the overgrown endometrium. c. Progesterone or combined oral contraceptives should be used to prevent persistent overgrowth of the endometrium. d. Unopposed progesterone production causes an overgrowth of the endometrium that increases her risk for endometrial cancer.

c. When ovulation does not occur, estrogen continues to be unopposed by progesterone and excessive buildup of the endometrium occurs. To prevent the risk of endometrial cancer by the buildup of the endometrium or to prevent heavy menstrual bleeding from an unstable endometrium, progesterone or combined oral contraceptives are prescribed to ensure regular shedding of the endometrial lining. Balloon therapy to treat heavy menstrual bleeding is contraindicated in women who desire future fertility and does not apply to amenorrhea.

A 20-yr-old woman is a college softball player who participates in strenuous practices and a heavy class schedule. She is describing an absence of menses. What could be contributing to her amenorrhea? a. Decreased sexual activity b. Excess prostaglandin production c. Strenuous exercise or elevated stress d. Endometrial cancer or uterine fibroids

c. Young female athletes may experience amenorrhea related to excessive exercise, low body weight, stress, or severe dieting. If she had increased sexual activity, she would be assessed for pregnancy, but decreased sexual activity will not affect her menses. Excess prostaglandin production leads to dysmenorrhea. Intermenstrual bleeding is associated with endometrial cancer or uterine fibroids.

*A patient is scheduled for a bone scan. The nurse explains that this diagnostic test involves* a. incision or puncture of the joint capsule. b. insertion of small needles into certain muscles. c. administration of a radioisotope before the procedure. d. placement of skin electrodes to record muscle activity

c. administration of a radioisotope before the procedure rationale: Bone scan involves injection of radioisotope (usually technetium [Tc]-99m) that is taken up by bone. Uniform uptake of the isotope is normal. Increased uptake is seen in osteomyelitis, primary and metastatic bone cancer, and certain fractures. Decreased uptake is seen in areas of avascular necrosis.

*In teaching a patient with Sjögren's syndrome about drug therapy for this disorder, the nurse includes instruction about the use of which drug?* a. Pregabalin (Lyrica) b. Etanercept (Enbrel) c. Cyclosporine (Restasis) d. Cyclobenzaprine (Flexeril)

c. cyclosporine (restasis) rationale: Cyclosporine (Restasis) ophthalmic drops can be used to treat the chronic dry eye associated with Sjögren's syndrome.

*The nurse suspects a neurovascular problem based on assessment of* a. exaggerated strength with movement. b. increased redness and heat below the injury. c. decreased sensation distal to the fracture site. d. purulent drainage at the site of an open fracture.

c. decreased sensation distal to the fracture site rationale: Musculoskeletal injuries can cause changes in the neurovascular condition of an injured extremity. The neurovascular assessment consists of peripheral vascular evaluation (e.g., color, temperature, capillary refill, peripheral pulses, and edema) and peripheral neurologic evaluation (e.g., sensation and motor function). Paresthesia and partial or full loss of sensation (paresis or paralysis) may be a sign of neurovascular damage. Pallor, a cool-to-cold extremity, or a delay in capillary refill time below the injury occur with arterial insufficiency. A decreased or absent pulse distal to the injury can indicate vascular dysfunction and insufficiency.

*A patient with a comminuted fracture of the tibia is to have an open reduction with internal fixation (ORIF) of the fracture. The nurse explains that ORIF is indicated when* a. the patient is unable to tolerate prolonged immobilization. b. the patient cannot tolerate the surgery for a closed reduction. c. other nonsurgical methods cannot achieve adequate alignment. d. a temporary cast would be too unstable to provide normal mobility.

c. other nonsurgical methods cannot achieve adequate alignment rationale: A comminuted fracture has more than 2 bone fragments. Open reduction with internal fixation (ORIF) is indicated for a comminuted fracture. It is used to realign and maintain bony fragments. Other nonsurgical methods can fail to obtain satisfactory reduction. Internal fixation reduces the hospital stay and complications associated with prolonged bed rest.

Patient-Centered Care: Following 7 days of IV antibiotic therapy, a patient with acute osteomyelitis of the tibia is prepared for discharge from the hospital. The nurse determines that additional instruction is needed when the patient makes which statement a. I will need to continue antibiotic therapy for 4 to 6 weeks b. I should notify the HCP if the pain in my leg becomes worse c. I shouldn't bear weight on my affected leg until healing is complete d. I do not need to do anything special while taking the antibiotic therapy

d

What does radicular pain that radiates down the buttock and below the knee, along the distribution of the sciatic nerve, generally indicate a. Cervical disc herniation b. acute lumboscaral strain c. Degenerative disc disease d. Herniated lumbar disc disease

d

Which type of bone tumor is a benign overgrowth of bone and cartilage and may transform into malignant form a. Enchondroma b. Osteoclastoma c. Ewing's sarcoma d. Ostochondroma

d

*An appropriate question to ask the patient with painful menstruation to differentiate primary from secondary dysmenorrhea is* a. "Does your pain become worse with activity or overexertion?" b. "Have you had a recent personal crisis or change in your lifestyle?" c. "Is your pain relieved by non steroidal anti-inflammatory medications?" d. "When in your menstrual history did the pain with your period begin?"

d.

13. A young patient with a fractured femur has a hip spica cast applied. What is the most important action the nurse should take while the cast is drying? a. Elevate the legs above the level of the heart for 24 hours. b. Turn the patient to both sides and prone to supine every 2 hours. c. Cover the cast with a light blanket to avoid chilling from evaporation. d. Assess the patient frequently for abdominal pain, nausea, and vomiting.

d. Abdominal pain or pressure, nausea, and vomiting are signs of cast syndrome that occur when hip spica casts or body jacket braces cause compression of the superior mesenteric artery against the duodenum because of swelling or tight application. The cast may have to be split or removed, and the HCP should be notified. Elevation is not indicated for a spica cast. The patient with a spica cast should not be placed in the prone position during the initial drying stage because the cast may break because it is so large and heavy. A cast should never be covered with a blanket because heat builds up in the cast and may cause tissue damage.

1. Patient-Centered Care: A 72-year-old man tells the nurse that he cannot perform most of the physical activities he could do 5 years ago because of overall joint aches and pains. What can the nurse do to help the patient prevent further deconditioning and decrease the risk for developing musculoskeletal problems? a. Limit weight-bearing exercise to prevent stress on fragile bones and possible hip fractures. b. Tell the patient to avoid using canes and walkers because they increase dependence on ambulation aids. c. Teach the patient to increase his activity by climbing stairs in buildings and other environments with steps. d. Discuss the use of stretching and strengthening exercises to decrease aches and pain so that exercise can be maintained.

d. Almost all older adults have some degree of decreased muscle strength, joint stiffness, and pain with motion. Warming up before and strengthening exercises help decrease aches and pains. Musculoskeletal problems in the older adult can be prevented with appropriate strategies, especially exercise. Walkers and canes should be used as necessary to decrease stress on joints so activity can be maintained. Stair walking can create enough stress on fragile bones to cause a hip fracture.

4. The patient works on a computer 8 hours each day. What kind of injury would the nurse assess for in this patient? a. Meniscus injury b. Rotator cuff injury c. Radial-ulnar fracture d. Carpal tunnel syndrome

d. Carpal tunnel syndrome would be expected related to continuous wrist movements. Injuries of the meniscus (fibrocartilage in the knee) are common with athletes. Radial-ulnar fractures are seen with great force, such as a fall or a car accident. Rotator cuff injuries occur with sudden adduction forces applied to the cuff while the arm is held in abduction. They are often seen with repetitive overhead motions.

30. What should the nurse include in discharge instructions for the patient after a hip prosthesis with a posterior approach? a. Restrict walking for 2 to 3 months. b. Take a bath rather than a shower to prevent falling. c. Keep the leg internally rotated while sitting and standing. d. Have a family member help the patient put on shoes and socks.

d. Having someone else put the patient's socks and shoes on for patients with hip prostheses with a posterior approach will protect the patient from extreme flexion, adduction, or internal rotation for at least 6 weeks to prevent dislocation of the prosthesis. Gradual weight bearing on the limb is allowed and ambulation should be encouraged. The leg should be not be internally rotated but kept in a neutral position.

A 29-yr-old woman is at the clinic with heavy menstrual bleeding. What finding would the nurse expect in this patient's assessment? a. Pain with each menstrual period b. Excessive bleeding at irregular intervals c. Bleeding between regular menstrual cycles d. Increased duration or amount of menstrual bleeding

d. Heavy menstrual bleeding is increased duration or amount of bleeding with menses. Pain with menses is called dysmenorrhea. Chronic abnormal uterine bleeding is classified as uterine bleeding that is abnormal in volume, timing, or regularity and has been present for most of the past 6 months. Intermenstrual bleeding is bleeding between regular menstrual cycles.

A patient asks the nurse what the difference is between benign prostatic hyperplasia (BPH) and prostate cancer. The best response by the nurse includes what information about BPH? a. BPH is a benign tumor that does not spread beyond the prostate gland. b. BPH is a precursor to prostate cancer but does not yet show any malignant changes. c. BPH is an enlargement of the gland caused by an increase in the size of existing cells. d. BPH is a benign enlargement of the gland caused by an increase in the number of normal cells.

d. Hyperplasia is an increase in the number of cells and in benign prostatic hyperplasia (BPH), it is thought that the enlargement caused by the increase in new cells results from hormonal changes associated with aging. Hypertrophy refers to an increase in the size of existing cells. The hyperplasia is not considered a tumor, nor has BPH been proven to predispose to cancer of the prostate.

A 66-year-old male patient is experiencing erectile dysfunction (ED). He and his wife have used tadalafil (Cialis) but because he experienced priapism, they have decided to change their treatment option to an intraurethral device. How should the nurse explain how this device works? a. The device relaxes smooth muscle in the penis. b. Blood is drawn into corporeal bodies and held with a ring. c. The device is implanted into corporeal bodies to firm the penis. d. The device directly applies drugs that increase blood flow in the penis.

d. Intraurethral devices include the use of vasoactive drugs administered as a topical gel, an injection into the penis (intracavernosal self-injection), or a medication pellet (alprostadil) inserted into the urethra (intraurethral) using a medicated urethral system for erection (MUSE) device. A medication pellet inserted into the urethra using a MUSE device, a topical gel, or the intracavernosal self-injection of vasoactive drugs may be used for erectile dysfunction. The vasoactive drugs enhance blood flow into the penile arteries for erection. Erectogenic drugs (e.g., tadalafil [Cialis]) cause smooth muscle relaxation and increase blood flow to promote an erection. Blood drawn into corporeal bodies and held with a ring is achieved with a vacuum constriction device (VCD). Devices implanted into corporeal bodies to firm the penis are penile implants. Androgen or testosterone replacement therapy may also be used for erectile dysfunction.

An 18-yr-old patient with irregular menstrual periods, hirsutism, and obesity has been diagnosed with polycystic ovary syndrome (PCOS). What is an accurate rationale for the expected treatment? a. Hirsutism may be treated with leuprolide to decrease an altered body image. b. The medication used will cure the hormonal abnormality of excess testosterone. c. The loss of weight will improve all of the symptoms, so this will be the first treatment tried. d. The progression of PCOS leads to cardiovascular disease and abnormal insulin resistance if untreated.

d. Left untreated, polycystic ovary syndrome (PCOS) may lead to cardiovascular disease and abnormal insulin resistance with type 2 diabetes mellitus. Hirsutism may be treated with spironolactone. Leuprolide is used to treat hyperandrogenism, but PCOS cannot be cured. Severity of symptoms is associated with obesity, but the hormone abnormalities will be treated along with the obesity to prevent complications. If other treatment is not successful, a hysterectomy with bilateral salpingectomy and oophorectomy may be performed.

What is an explanation that the nurse should give to the patient who asks what his diagnosis of paraphimosis means? a. Painful, prolonged erection b. Inflammation of the epididymis c. Painful downward curvature of an erect penis d. Retracted tight foreskin preventing return over the glans

d. Paraphimosis is tightness of the foreskin and the inability to pull it forward from a retracted position to return it over the glans. It is usually associated with poor hygiene techniques. Painful, prolonged erection is priapism. Epididymitis is inflammation of the epididymis. A painful downward curvature of an erect penis is chordee.

39. A patient had a right total hip arthroplasty with a cemented prosthesis for treatment of severe osteoarthritis of the hip. What activity should the nurse include on the patient's first postoperative day? a. Transfer from the bed to the chair twice a day only b. Only turning from the back to both sides every 2 hours c. Crutch walking with non-weight bearing on the operative leg d. Ambulation with a walker and limited weight bearing on the right leg

d. Physical therapy is initiated on the first postoperative day with ambulation using a walker and limited weight bearing for a patient with a cemented prosthesis and non-weight-bearing on the operative side for an uncemented prosthesis. In addition, the patient sits in the chair at least twice a day and is turned to the back and unaffected side with the operative leg supported. Crutches would be difficult to use the first postoperative day.

Premenstrual syndrome (PMS) is most likely to be diagnosed in a woman with which occurrence? a. Symptoms can be controlled with the use of progesterone. b. The woman has symptoms only when oral contraceptives are used. c. Symptoms can be correlated with altered serum levels of estrogen and progesterone. d. The woman has consistent syndrome complex with symptoms ending after menses begins.

d. Premenstrual syndrome (PMS) is diagnosed when other possible causes for symptoms have been eliminated. A diagnosis is based on (1) consistency of the syndrome complex, (2) occurrence of symptoms in the luteal phase and resolution after menses begins, (3) documented ovulatory cycles, and (4) symptoms that disrupt the woman's life. Oral contraceptives may be used to control the symptoms of PMS by suppressing ovulation, and although progesterone may also relieve the symptoms of PMS, its effectiveness is not associated with the diagnosis of PMS. There are no laboratory findings that account for the premenstrual symptoms.

15. Priority Decision: A patient who fell in the bathroom of the hospital room reports pain in the upper right arm and elbow. Which action should the nurse take first in managing a possible fracture before splinting the injury? a. Elevate the arm b. Apply ice to the site c. Notify the health care provider d. Perform a neurovascular check below the injury

d. Sensation, motor function, and pain distal to the injury should be checked before and after splinting to assess for nerve damage. Document results to avoid doubts about whether a problem discovered later was missed during the original examination or was caused by the treatment. Peripheral vascular assessment is needed and the HCP is notified. Elevation of the limb and application of ice should be instituted after the extremity is splinted.

When teaching a patient testicular self-examination, the nurse instructs the patient to report which finding? a. An irregular-feeling epididymis b. One testis larger than the other c. The spermatic cord within the testicle d. A firm, nontender nodule on the testis

d. Testicular tumors most often present on the testis as a lump or nodule that is very firm, is nontender, and cannot be transilluminated. There may also be scrotal swelling and a feeling of heaviness. All of the other options are normal findings.

*When grading muscle strength, the nurse records a score of 3/5, which indicates* a. no detection of muscular contraction. b. a barely detectable flicker of contraction. c. active movement against full resistance without fatigue. d. active movement against gravity but not against resistance

d. active movement against gravity but not against resistance rationale: Muscle strength score of 3 indicates active movement only against gravity and not against resistance (Table 61-4).

27. An older adult woman is admitted to the ED after falling at home. The nurse cautions her not to put weight on the leg after finding what in the patient assessment? a. Inability to move the toes and ankle b. Edema of the thigh extending to the knee c. Internal rotation of the leg with groin pain d. Shortening and external rotation of the leg

d. The classic signs of a hip fracture are external rotation and shortening of the leg accompanied by severe pain at the fracture site. Additional injury could be caused by weight bearing on the extremity. The patient may not be able to move the hip or the knee, but movement in the ankle and toes is not affected.

The cremasteric reflex is absent in which problem of the scrotum and testes? a. Hydrocele b. Varicocele c. Spermatocele d. Testicular torsion

d. The cremasteric reflex is elicited by light stroking of the inner aspect of the thigh in a downward direction with a tongue blade. In testicular torsion, or a twisted spermatic cord that supplies blood to the testes and epididymis, this reflex is absent on the swollen side. Varicocele is dilation of the veins that drain the testes. Hydrocele is scrotal lymphedema from interference with lymphatic drainage of the scrotum. Spermatocele is a sperm-containing cyst of the epididymis.

43. Priority Decision: After change-of-shift handoff, which patient should the nurse assess first? a. A 58-year-old male reporting phantom pain and requesting an analgesic b. A 72-year-old male being transferred to a skilled nursing unit after repair of a hip fracture c. A 25-year-old female in left leg skeletal traction asking for the weights to be lifted for a few minutes d. A 68-year-old male with a new lower leg cast reporting the cast is too tight and he cannot feel his toes

d. The patient with a tight cast may be at risk for neurovascular compromise (impaired circulation and peripheral nerve damage) and should be assessed first. The other patients should be seen as soon as possible. Providing analgesia for the patient with phantom pain would be the next priority. The patient in skeletal traction needs explanation of the purpose and functioning of the traction. She may need analgesia or muscle relaxants to help tolerate the traction. Checking on the patient being transferred would include reassurance and paperwork completion.

Priority Decision: The patient's diagnosis is a large rectocele requiring surgery. What nursing interventions will be the priority postoperatively? a. An ice pack to relieve swelling b. An enema each day to relive constipation c. Administration of a stool softener each night d. Perineal care after each urination or defecation

d. The primary goal of care is to prevent wound infection and pressure on the vaginal incision, which requires perineal cleansing at least twice daily and after each urination and defecation. An ice pack and stool softener will be used, but they are not the priority. The enema would have been done preoperatively.

What is the most common screening intervention for detecting BPH in men over age 50? a. PSA level b. Urinalysis c. Cystoscopy d. Digital rectal examination

d. The prostate gland can be easily palpated by rectal examination and enlargement of the gland is detected early if yearly examinations are performed. If symptoms of prostatic hyperplasia are present, further diagnostic testing, including a urinalysis, prostate-specific antigen (PSA), and cystoscopy, may be indicated.

What occurs with chronic bacterial prostatitis but not with acute prostatitis? a. Postejaculatory pain b. Frequency, urgency, and dysuria c. Symptoms of a urinary tract infection d. Enlarged, boggy prostate on palpation

d. The prostate with chronic bacterial prostatitis feels enlarged and firm, often described as boggy, and is tender. The other options are true of both chronic and acute prostatitis.

The patient is a perimenopausal woman who has an unexpected and unwanted pregnancy. She wants an abortion. What should the nurse teach her about the effects of an abortion? a. D&C will be needed. b. She will feel much better afterward. c. The products of conception will pass immediately. d. She will need someone to support her through her loss.

d. There is physical and emotional pain and grieving after an abortion that puts the patient in need of support. D&C is needed only if the products of conception do not pass completely or bleeding becomes excessive. The time it takes for the products of conception to pass depends on the type of abortion being done and is immediate with surgical abortion and slower with medical abortion.

What occurs during treatment of the patient with an intrauterine radioactive implant? a. All care should be provided by the same nurse. b. The patient may ambulate in the room as desired. c. There can be unlimited number and duration of visitors. d. The patient is restricted to bed rest with turning from side to side

d. To prevent displacement of the intrauterine implant, the patient is maintained on absolute bed rest with turning from side to side. Bowel elimination is discouraged during the treatment by cleaning the colon before implantation and urinary elimination is maintained by an indwelling catheter. Because the patient is radioactive, the principles of ALARA (as low as reasonably achievable) are used so that caregivers limit time and distance and use shielding for protection.

The couple has not been able to become pregnant. The wife has not been diagnosed with any infertility problems. Which treatment will the nurse expect to teach the couple about if the problem is the most common testicular problem causing male infertility? a. Antibiotics b. Semen analysis c. Avoidance of scrotal heat d. Surgery to correct the problem

d. Varicocele is the most common testicular cause of infertility. Surgical ligation of the spermatic vein is done to correct the problem. Antibiotics are used if there is an infection but this is not as common as a varicocele. Semen analysis is the first study done when investigating male infertility but it is not a treatment. Avoidance of scrotal heat is a lifestyle change that may be used with idiopathic infertility.

*A patient is scheduled for total ankle replacement. The nurse should tell the patient that after surgery he should avoid* a. lifting heavy objects. b. sleeping on the back. c. abduction exercises of the affected ankle. d. bearing weight on the affected leg for 6 weeks.

d. bearing weight on the affected leg for 6 weeks rationale: After total ankle arthroplasty (TAA), the patient may not bear weight for 6 weeks and must elevate the extremity to reduce edema. The patient must follow strategies to prevent postoperative infection and maintain immobilization as directed by the provider.

An older patient is describing increased rigidity in the shoulders back and hips. The loss of elasticity in what tissue contributes to this a. actin b. fascia c. myosin d. ligament

d. ligament

*The bone cells that function in the formation of new bone tissue are called* a. osteoids. b. osteocytes. c. osteoclasts. d. osteoblasts.

d. osteoblasts rationale: Osteoblasts are the main cells that form new bone.

*A patient with a stable, closed humeral fracture has a temporary splint with bulky padding applied with an elastic bandage. The nurse notifies the provider of possible early compartment syndrome when the patient has* a. increasing edema of the limb. b. muscle spasms of the lower arm. c. bounding pulse at the fracture site. d. pain when passively extending the fingers.

d. pain when passively extending the fingers rationale: One or more of these characteristics occur with early compartment syndrome: (1) paresthesia (e.g., numbness and tingling sensation); (2) pain distal to the injury that is not relieved by opioid analgesics and is increased on passive stretch of muscle; (3) increased pressure in the compartment; and (4) pallor, coolness, and loss of normal color of the extremity. Paralysis (or loss of function) and pulselessness (or decreased or absent peripheral pulses) are late signs of compartment syndrome. The examination also includes assessment of peripheral edema, especially pitting edema, which may occur with severe injury

*A patient with suspected disc herniation has acute pain and muscle spasms. The nurse's responsibility is to* a. encourage total bed rest for several days. b. teach principles of back strengthening exercises. c. stress the importance of straight-leg raises to decrease pain. d. promote use of cold and hot compresses and pain medication.

d. promote use of cold and hot compresses and pain medication rationale: If acute pain and muscle spasms are not severe, the patient may be treated as an outpatient with NSAIDs and muscle relaxants (e.g., cyclobenzaprine). Massage and back manipulation, acupuncture, and application of cold and hot compresses may help some patients. A brief period (1 to 2 days) of rest at home may be needed for some people; most patients do better if they continue their regular activities. Prolonged bed rest should be avoided. All patients should refrain from activities that worsen the pain, including lifting, bending, twisting, and prolonged sitting.

*In teaching a patient with systemic lupus erythematosus about the disorder, the nurse knows the pathophysiology includes* a. circulating immune complexes formed from IgG autoantibodies reacting with IgG. b. an autoimmune T-cell reaction that results in destruction of the deep dermal skin layer. c. immunologic dysfunction leading to chronic inflammation in the cartilage and muscles. d. the production of a variety of autoantibodies directed against components of the cell nucleus.

d. the production of a variety of autoantibodies directed against components of the cell nucleus rationale: SLE is marked by production of many autoantibodies against nucleic acids (e.g., single-and double-stranded DNA), erythrocytes, coagulation proteins, lymphocytes, platelets, and many other self-proteins. Autoimmune reactions (antinuclear antibodies [ANA]) are typically directed against constituents of the cell nucleus, especially DNA.

*The nurse suspects an ankle sprain when a patient at the urgent care center describes* a. being hit by another soccer player during a game. b. having ankle pain after sprinting around the track. c. dropping a 10-lb weight on his lower leg at the health club. d. twisting his ankle while running bases during a baseball game.

d. twisting his ankle while running bases during a baseball game. rationale: A sprain is an injury to the ligaments surrounding a joint. It is usually caused by a wrenching or twisting motion. Most sprains occur in the ankle and knee joints

Which characteristics describe transurethral resection of the prostate (TURP) (select all that apply)? a. Best used for a very large prostate gland b. Inappropriate for men with rectal problems c. Involves an external incision prostatectomy d. Uses transurethral incisions into the prostate e. Most common surgical procedure to treat BPH f. Resectoscopic excision and cauterization of prostate tissue

e, f. The transurethral resection of the prostate (TURP) is the most common surgical procedure to treat BPH and uses a resectoscopic excision and cauterization of prostate tissue. A simple open prostatectomy is used for a large prostate and has an external incision. Transurethral incision into the prostate to expand the urethra for a small to moderate-sized prostate is done with a transurethral incision of the prostate (TUIP).

A patient has been diagnosed with cancer of the ovary. In planning care for the patient, the nurse recognizes that treatment of the patient depends on what? a. Results of a direct-needle biopsy of the ovary b. Results of a laparotomy with multiple biopsies c. Whether the patient desires to maintain fertility d. The findings of metastasis by ultrasound or CT scan

from the results of laparotomy with multiple biopsies of the ovaries and other tissue throughout the pelvis and lower abdomen. The patient's desire for fertility is not a consideration because of the high mortality rate associated with ovarian cancer. Although diagnosis of ovarian tumors may be made by transvaginal ultrasound or CT scan, the treatment of ovarian cancer depends on the staging of the tumor.


Conjuntos de estudio relacionados

C1 Campigns and Election indirect direct primary open closed case Citizens United v. Federal Election Commission 2 and att

View Set

Government 2305 Chapter 2 Quizzes

View Set

MKTG ch 3, Marketing 351 Chapter 1, Marketing, MKTG Chapter 1, MKTG Chapter 5, MKT 230 - chp. 5

View Set